Решение системных уравнений методом гаусса онлайн: Онлайн калькулятор. Решение систем линейных уравнений. Метод Гаусса.

Содержание

принцип, теорема и примеры решения задач

Задание. Решить СЛАУ $\left\{\begin{array}{l} 2 x_{1}+x_{2}+x_{3}=2 \\ x_{1}-x_{2}=-2 \\ 3 x_{1}-x_{2}+2 x_{3}=2 \end{array}\right.$ методом Гаусса.

Решение. Выпишем расширенную матрицу системы и при помощи элементарных преобразований над ее строками приведем эту матрицу к ступенчатому виду (прямой ход) и далее выполним обратный ход метода Гаусса (сделаем нули выше главной диагонали). Вначале поменяем первую и вторую строку, чтобы элемент $a_{11}$ равнялся 1 (это мы делаем для упрощения вычислений):

$$\tilde{A}=A \mid B=\left(\begin{array}{rrr|r} 2 & 1 & 1 & 2 \\ 1 & -1 & 0 & -2 \\ 3 & -1 & 2 & 2 \end{array}\right) \sim\left(\begin{array}{rrr|r} 1 & -1 & 0 & -2 \\ 2 & 1 & 1 & 2 \\ 3 & -1 & 2 & 2 \end{array}\right)$$

Далее делаем нули под главной диагональю в первом столбце.

Для этого от второй строки отнимаем две первых, от третьей – три первых:

$$\tilde{A} \sim\left(\begin{array}{rrr|r} 1 & -1 & 0 & -2 \\ 0 & 3 & 1 & 6 \\ 0 & 2 & 2 & 8 \end{array}\right)$$

Все элементы третьей строки делим на два (или, что тоже самое, умножаем на $\frac{1}{2}$ ):

$$\tilde{A} \sim\left(\begin{array}{rrr|r} 1 & -1 & 0 & -2 \\ 0 & 3 & 1 & 6 \\ 0 & 1 & 1 & 4 \end{array}\right)$$

Далее делаем нули во втором столбце под главной диагональю, для удобства вычислений поменяем местами вторую и третью строки, чтобы диагональный элемент равнялся 1:

$$\tilde{A} \sim\left(\begin{array}{ccc|c} 1 & -1 & 0 & -2 \\ 0 & 1 & 1 & 4 \\ 0 & 3 & 1 & 6 \end{array}\right)$$

От третьей строки отнимаем вторую, умноженную на 3:

$$\tilde{A} \sim\left(\begin{array}{rrr|r} 1 & -1 & 0 & -2 \\ 0 & 1 & 1 & 4 \\ 0 & 0 & -2 & -6 \end{array}\right)$$

Умножив третью строку на $\left(-\frac{1}{2}\right)$ , получаем:

$$\tilde{A} \sim\left(\begin{array}{rrr|r} 1 & -1 & 0 & -2 \\ 0 & 1 & 1 & 4 \\ 0 & 0 & 1 & 3 \end{array}\right)$$

Проведем теперь обратный ход метода Гаусса (метод Гассу-Жордана), то есть сделаем нули над главной диагональю. Начнем с элементов третьего столбца. Надо обнулить элемент $a_{23}$, для этого от второй строки отнимем третью:

$$\tilde{A} \sim\left(\begin{array}{rrr|r} 1 & -1 & 0 & -2 \\ 0 & 1 & 0 & 1 \\ 0 & 0 & 1 & 3 \end{array}\right)$$

Далее обнуляем недиагональные элементы второго столбца, к первой строке прибавляем вторую:

$$\tilde{A} \sim\left(\begin{array}{ccc|c} 1 & 0 & 0 & -1 \\ 0 & 1 & 0 & 1 \\ 0 & 0 & 1 & 3 \end{array}\right)$$

Полученной матрице соответствует система

$\left\{\begin{array}{l}x_{1}+0 \cdot x_{2}+0 \cdot x_{3}=-1 \\ 0 \cdot x_{1}+x_{2}+0 \cdot x_{3}=1 \\ 0 \cdot x_{1}+0 \cdot x_{2}+x_{3}=3\end{array}\right.$    или   $\left\{\begin{array}{l} x_{1}=-1 \\ x_{2}=1 \\ x_{3}=3 \end{array}\right.$

Ответ. $\left\{\begin{array}{l} x_{1}=-1 \\ x_{2}=1 \\ x_{3}=3 \end{array}\right.$

Решение уравнений методом сложения.

Калькулятор онлайн

Соблюдение Вашей конфиденциальности важно для нас. По этой причине, мы разработали Политику Конфиденциальности, которая описывает, как мы используем и храним Вашу информацию. Пожалуйста, ознакомьтесь с нашими правилами соблюдения конфиденциальности и сообщите нам, если у вас возникнут какие-либо вопросы.

Сбор и использование персональной информации

Под персональной информацией понимаются данные, которые могут быть использованы для идентификации определенного лица либо связи с ним.

От вас может быть запрошено предоставление вашей персональной информации в любой момент, когда вы связываетесь с нами.

Ниже приведены некоторые примеры типов персональной информации, которую мы можем собирать, и как мы можем использовать такую информацию.

Какую персональную информацию мы собираем:

  • Когда вы оставляете заявку на сайте, мы можем собирать различную информацию, включая ваши имя, номер телефона, адрес электронной почты и т.д.

Как мы используем вашу персональную информацию:

  • Собираемая нами персональная информация позволяет нам связываться с вами и сообщать об уникальных предложениях, акциях и других мероприятиях и ближайших событиях.
  • Время от времени, мы можем использовать вашу персональную информацию для отправки важных уведомлений и сообщений.
  • Мы также можем использовать персональную информацию для внутренних целей, таких как проведения аудита, анализа данных и различных исследований в целях улучшения услуг предоставляемых нами и предоставления Вам рекомендаций относительно наших услуг.
  • Если вы принимаете участие в розыгрыше призов, конкурсе или сходном стимулирующем мероприятии, мы можем использовать предоставляемую вами информацию для управления такими программами.

Раскрытие информации третьим лицам

Мы не раскрываем полученную от Вас информацию третьим лицам.

Исключения:

  • В случае если необходимо – в соответствии с законом, судебным порядком, в судебном разбирательстве, и/или на основании публичных запросов или запросов от государственных органов на территории РФ – раскрыть вашу персональную информацию. Мы также можем раскрывать информацию о вас если мы определим, что такое раскрытие необходимо или уместно в целях безопасности, поддержания правопорядка, или иных общественно важных случаях.
  • В случае реорганизации, слияния или продажи мы можем передать собираемую нами персональную информацию соответствующему третьему лицу – правопреемнику.

Защита персональной информации

Мы предпринимаем меры предосторожности – включая административные, технические и физические – для защиты вашей персональной информации от утраты, кражи, и недобросовестного использования, а также от несанкционированного доступа, раскрытия, изменения и уничтожения.

Соблюдение вашей конфиденциальности на уровне компании

Для того чтобы убедиться, что ваша персональная информация находится в безопасности, мы доводим нормы соблюдения конфиденциальности и безопасности до наших сотрудников, и строго следим за исполнением мер соблюдения конфиденциальности.

ОГБОУ «Центр образования для детей с особыми образовательными потребностями г. Смоленска»

Центр дистанционного образования

Урок алгебры в 7 классе

Тема урока: Метод алгебраического сложения.

      1. Тип урока: Урок первичного предъявления новых знаний.

Цель урока: контроль уровня усвоения знаний и умений решения систем уравнений способом подстановки; формирование умений и навыков решения систем уравнений способом сложения.

Задачи урока:

Предметные: научиться выполнять решения систем уравнений с двумя переменными методом сложения.

Метапредметные: Познавательные УУД : анализировать (выделять главное), определять понятия, обобщать, делать выводы. Регулятивные УУД : определять цель, проблему в учебной деятельности. Коммуникативные УУД : излагать своё мнение, аргументируя его. Личностные УУД: ф ормировать положительную мотивацию к обучению, создавать позитивное эмоциональное отношение обучающегося к уроку и предмету.

Форма работы: индивидуальная

Этапы урока:

1) Организационный этап.

организовать работу обучающейся по теме через создание установки на целостность мышления и понимание данной темы.

2. Опрос обучающейся по заданному на дом материалу, актуализация знаний.

Цель: проверить знания обучающейся, полученные в ходе выполнения домашней работы, выявить ошибки, сделать работу над ошибками. Повторить материал прошлого урока.

3. Изучение нового материала.

1). формировать умение решать системы линейных уравнений способом сложения;

2). развивать и совершенствовать имеющиеся знания в новых ситуациях;

3). воспитывать навыки контроля и самоконтроля, развивать самостоятельность.

http://zhakulina20090612.blogspot.ru/2011/06/blog-post_25.html

Цель: сохранение зрения, снятие усталости с глазво время работы на уроке.

5. Закрепление изученного материала

Цель: проверить знания, умения и навыки, полученные на уроке

6. Итог урока, информация о домашнем задании, рефлексия.

Ход урока (работа в электронном документе Google):

1. Сегодня урок я хотела начать с философской загадки Вальтера.

Что самое быстрое, но и самое медленное, самое большое, но и самое маленькое, самое продолжительное и короткое, самое дорогое, но и дешево ценимое нами?

Время

Вспомним основные понятия по теме:

Перед нами система двух уравнений.

Вспомним, как мы решали системы уравнений на прошлом уроке.

Методом подстановки

Еще раз обрати внимание на решенную систему и скажи, почему мы не можем решить каждое уравнение системы не прибегая к методу подстановки?

Потому что это – уравнения системы с двумя переменными. Мы умеем решать уравнение только с одной переменной.

Только получив уравнение с одной переменной нам удалось решить систему уравнений.

3. Мы приступаем к решению следующей системы:

Выберем уравнение, в котором удобно одну переменную выразить через другую.

Такого уравнения нет.

Т.е. в данной ситуации нам не подходит изученный ранее метод.

Какой выход из данной ситуации?

Найти новый метод.

Попытаемся сформулировать цель урока.

Научиться решать системы новым методом.

Что нам необходимо сделать, чтобы научиться решать системы новым методом?

знать правила (алгоритм) решения системы уравнения, выполнить практические задания

Приступим к выведению нового метода.

Обрати внимание на вывод, который мы сделали после решения первой системы. Решить систему удалось только после того, как мы получили линейное уравнение с одной переменной.

Посмотри на систему уравнений и подумай, как из двух данных уравнений получить одно уравнение с одной переменной.

Сложить уравнения.

Что значит сложить уравнения?

По отдельности составить сумму левых частей, сумму правых частей уравнений и полученные суммы приравнять.

Попробуем. Работаем вместе со мной.

13x+14x+17y-17y=43+11

Получили линейное уравнение с одной переменной.

Решили систему уравнений?

Решение системы – пара чисел.

Как найти у?

Найденное значение х подставить в уравнение системы.

Имеет значение, в какое уравнение подставим значение х?

Значит найденное значение х можно подставить в…

любое уравнение системы.

Мы познакомились с новым методом – методом алгебраического сложения.

Решая систему, мы проговорили алгоритм решения системы данным методом.

Алгоритм мы рассмотрели. Теперь применим его к решению задач.

Умение решать системы уравнений может пригодится в практике.

Рассмотрим задачу:

В хозяйстве имеются куры и овцы. Сколько тех и других, если у них вместе 19 голов и 46 ног?

Зная, что всего кур и овец 19, составим первое уравнение: х + у =19

4х – число ног у овец

2у – число ног у кур

Зная, что всего 46 ног, составим второе уравнение: 4х + 2у =46

Составим систему уравнений:

Решим систему уравнений, применяя алгоритм решения методом сложения.

Проблема! Коэффициенты перед х и у – не равные и не противоположные! Что же делать?

Рассмотрим ещё один пример!

Добавим в наш алгоритм ещё один шаг и поставим его на первое место: Если коэффициенты перед переменными- не одинаковые и не противоположные, то надо уравнять модули при какой-нибудь переменной! А далее уже будем действовать по алгоритму.

4. Электронная физкультминутка для глаз: http://zhakulina20090612.blogspot.ru/2011/06/blog-post_25.html

5. Дорешаем задачу методом алгебраического сложения, закрепив новый материал и узнаем, сколько же кур и овец было в хозяйстве.

Дополнительные задания:

6.

Рефлексия.

Я за свою работу на уроке ставлю оценку – …

6. Использованные ресурсы-интернет:

сервисы Google для образования

Учитель математики Соколова Н. Н.

Системой линейных уравнений с двумя неизвестными – это два или несколько линейных уравнений, для которых необходимо найти все их общие решения. Мы будем рассматривать системы из двух линейных уравнений с двумя неизвестными. Общий вид системы из двух линейных уравнений с двумя неизвестными представлен на рисунке ниже:

{ a1*x + b1*y = c1,
{ a2*x + b2*y = c2

Здесь х и у неизвестные переменные, a1,a2,b1,b2,с1,с2 – некоторые вещественные числа. Решением системы двух линейных уравнений с двумя неизвестными называют пару чисел (x,y) такую, что если подставить эти числа в уравнения системы, то каждое из уравнений системы обращается в верное равенство. Существует несколько способов решения системы линейных уравнений. Рассмотрим один из способов решения системы линейных уравнений, а именно способ сложения.

Алгоритм решения способом сложения

Алгоритм решения системы линейных уравнений с двумя неизвестными способом сложения.

1. Если требуется, путем равносильных преобразований уравнять коэффициенты при одной из неизвестных переменных в обоих уравнениях.

2. Складывая или вычитая полученные уравнения получить линейное уравнение с одним неизвестным

3. Решить полученное уравнение с одним неизвестным и найти одну из переменных.

4. Подставить полученное выражение в любое из двух уравнений системы и решить это уравнение, получив, таким образом, вторую переменную.

5. Сделать проверку решения.

Пример решения способом сложения

Для большей наглядности решим способом сложения следующую систему линейных уравнений с двумя неизвестными:

{3*x + 2*y = 10;
{5*x + 3*y = 12;

Так как, одинаковых коэффициентов нет ни у одной из переменных, уравняем коэффициенты у переменной у. Для этого умножим первое уравнение на три, а второе уравнение на два.

{3*x+2*y=10 |*3
{5*x + 3*y = 12 |*2

Получим следующую систему уравнений:

{9*x+6*y = 30;
{10*x+6*y=24;

Теперь из второго уравнения вычитаем первое. Приводим подобные слагаемые и решаем полученное линейное уравнение.

10*x+6*y – (9*x+6*y) = 24-30; x=-6;

Полученное значение подставляем в первое уравнение из нашей исходной системы и решаем получившееся уравнение.

{3*(-6) + 2*y =10;
{2*y=28; y =14;

Получилась пара чисел x=6 и y=14. Проводим проверку. Делаем подстановку.

{3*x + 2*y = 10;
{5*x + 3*y = 12;

{3*(-6) + 2*(14) = 10;
{5*(-6) + 3*(14) = 12;

{10 = 10;
{12=12;

Как видите, получились два верных равенства, следовательно, мы нашли верное решение.

Методом сложения, уравнения системы почленно складывают, при этом 1-но либо оба (несколько) уравнений можно умножить на любое число. В результате приходят к равнозначной СЛУ , где в одном из уравнений есть лишь одна переменная.

Для решения системы способом почленного сложения (вычитания) следуйте следующим шагам:

1. Выбираем переменную, у которой будут делаться одинаковые коэффициенты.

2. Теперь нужно сложить либо вычесть уравнения и получим уравнение с одной переменной.

Решение системы – это точки пересечения графиков функции.

Рассмотрим на примерах.

Пример 1.

Дана система:

Проанализировав эту систему можно заметить, что коэффициенты при переменной равны по модулю и разные по знаку (-1 и 1). В таком случае уравнения легко сложить почленно:

Действия, которые обведены красным цветом, выполняем в уме.

Результатом почленного сложения стало исчезновение переменной y . Именно в этом и В этом, собственно, и заключается смысл метода – избавиться от 1-ой из переменных.

-4 – y + 5 = 0 → y = 1,

В виде системы решение выглядит где-то так:

Ответ: x = -4 , y = 1.

Пример 2.

Дана система:

В этом примере можете пользоваться «школьным» методом, но в нем есть немаленький минус – когда вы будете выражать любую переменную из любого уравнения, то получите решение в обыкновенных дробях . А решение дробей занимает достаточно времени и вероятность допущения ошибок увеличивается.

Поэтому лучше пользоваться почленным сложением (вычитанием) уравнений. Проанализируем коэффициенты у соответствующих переменных:

Нужно подобрать число, которое можно поделить и на 3 и на 4 , при этом нужно, что бы это число было минимально возможным. Это наименьшее общее кратное . Если вам тяжело подобрать подходящее число, то можете перемножить коэффициенты: .

Следующий шаг:

1-е уравнение умножаем на ,

3-е уравнение умножаем на ,

Системы уравнений получили широкое применение в экономической отрасли при математическом моделировании различных процессов. Например, при решении задач управления и планирования производства, логистических маршрутов (транспортная задача) или размещения оборудования.

Системы уравнения используются не только в области математики, но и физики, химии и биологии, при решении задач по нахождению численности популяции.

Системой линейных уравнений называют два и более уравнения с несколькими переменными, для которых необходимо найти общее решение. Такую последовательность чисел, при которых все уравнения станут верными равенствами или доказать, что последовательности не существует.

Линейное уравнение

Уравнения вида ax+by=c называют линейными. Обозначения x, y – это неизвестные, значение которых надо найти, b, a – коэффициенты при переменных, c – свободный член уравнения.
Решение уравнение путем построение его графика будет иметь вид прямой, все точки которой являются решением многочлена.

Виды систем линейных уравнений

Наиболее простыми считаются примеры систем линейных уравнений с двумя переменными X и Y.

F1(x, y) = 0 и F2(x, y) = 0, где F1,2 – функции, а (x, y) – переменные функций.

Решить систему уравнений это значит найти такие значения (x, y), при которых система превращается в верное равенство или установить, что подходящих значений x и y не существует.

Пара значений (x, y), записанная в виде координат точки, называется решением системы линейных уравнений.

Если системы имеют одно общее решение или решения не существует их называют равносильными.

Однородными системами линейных уравнений являются системы правая часть которых равна нулю. Если правая после знака “равенство” часть имеет значение или выражена функцией, такая система неоднородна.

Количество переменных может быть гораздо больше двух, тогда следует говорить о примере системы линейных уравнений с тремя переменными или более.

Сталкиваясь с системами школьники предполагают, что количество уравнений обязательно должно совпадать с количеством неизвестных, но это не так. Количество уравнений в системе не зависит от переменных, их может быть сколь угодно много.

Простые и сложные методы решения систем уравнений

Не существует общего аналитического способа решения подобных систем, все методы основаны на численных решениях. В школьном курсе математики подробно описаны такие методы как перестановка, алгебраическое сложение, подстановка, а так же графический и матричный способ, решение методом Гаусса.

Основная задача при обучении способам решения – это научить правильно анализировать систему и находить оптимальный алгоритм решения для каждого примера. Главное не вызубрить систему правил и действий для каждого способа, а понять принципы применения того или иного метода

Решение примеров систем линейных уравнений 7 класса программы общеобразовательной школы довольно простое и объяснено очень подробно. В любом учебнике математике этому разделу отводится достаточно внимания. Решение примеров систем линейных уравнений методом Гаусса и Крамера более подробно изучают на первых курсах высших учебных заведений.

Решение систем методом подстановки

Действия метода подстановки направлены на выражение значения одной переменной через вторую. Выражение подставляется в оставшееся уравнение, затем его приводят к виду с одной переменной. Действие повторяется в зависимости от количества неизвестных в системе

Приведем решение примера системы линейных уравнений 7 класса методом подстановки:

Как видно из примера, переменная x была выражена через F(X) = 7 + Y. Полученное выражение, подставленное во 2-е уравнение системы на место X, помогло получить одну переменную Y во 2-е уравнении. Решение данного примера не вызывает трудностей и позволяет получить значение Y. Последний шаг это проверка полученных значений.

Решить пример системы линейных уравнений подстановкой не всегда возможно. Уравнения могут быть сложными и выражение переменной через вторую неизвестную окажется слишком громоздким для дальнейших вычислений. Когда неизвестных в системе больше 3-х решение подстановкой также нецелесообразно.

Решение примера системы линейных неоднородных уравнений:

Решение с помощью алгебраического сложения

При поиске решении систем методом сложения производят почленное сложение и умножение уравнений на различные числа. Конечной целью математических действий является уравнение с одной переменной.

Для применений данного метода необходима практика и наблюдательность. Решить систему линейных уравнений методом сложения при количестве переменных 3 и более непросто. Алгебраическое сложение удобно применять когда в уравнениях присутствуют дроби и десятичные числа.

Алгоритм действий решения:

  1. Умножить обе части уравнения на некое число. В результате арифметического действия один из коэффициентов при переменной должен стать равным 1.
  2. Почленно сложить полученное выражение и найти одно из неизвестных.
  3. Подставить полученное значение во 2-е уравнение системы для поиска оставшейся переменной.

Способ решения введением новой переменной

Новую переменную можно вводить, если в системе требуется найти решение не более чем для двух уравнений, количество неизвестных тоже должно быть не больше двух.

Способ используется, чтобы упростить одно из уравнений, вводом новой переменной. Новое уравнение решается относительно введенной неизвестной, а полученное значение используется для определения первоначальной переменной.

Из примера видно, что введя новую переменную t удалось свести 1-е уравнение системы к стандартному квадратному трехчлену. Решить многочлен можно отыскав дискриминант.

Необходимо найти значение дискриминанта по известной формуле: D = b2 – 4*a*c, где D – искомый дискриминант, b, a, c – множители многочлена. В заданном примере a=1, b=16, c=39, следовательно, D=100. Если дискриминант больше нуля, то решений два: t = -b±√D / 2*a, если дискриминант меньше нуля, то решение одно: x= -b / 2*a.

Решение для полученных в итоге системы находят методом сложения.

Наглядный метод решения систем

Подходит для систем с 3-мя уравнениями. Метод заключается в построении на координатной оси графиков каждого уравнения, входящего в систему. Координаты точек пересечения кривых и будут общим решением системы.

Графический способ имеет ряд нюансов. Рассмотрим несколько примеров решения систем линейных уравнений наглядным способом.

Как видно из примера, для каждой прямой было построено две точки, значения переменной x были выбраны произвольно: 0 и 3. Исходя из значений x, найдены значения для y: 3 и 0. Точки с координатами (0, 3) и (3, 0) были отмечены на графике и соединены линией.

Действия необходимо повторить для второго уравнения. Точка пересечения прямых является решением системы.

В следующем примере требуется найти графическое решение системы линейных уравнений: 0,5x-y+2=0 и 0,5x-y-1=0.

Как видно из примера, система не имеет решения, потому что графики параллельны и не пересекаются на всем своем протяжении.

Системы из примеров 2 и 3 похожи, но при построении становится очевидно, что их решения разные. Следует помнить, что не всегда можно сказать имеет ли система решение или нет, всегда необходимо построить график.

Матрица и ее разновидности

Матрицы используются для краткой записи системы линейных уравнений. Матрицей называют таблицу специального вида, заполненную числами. n*m имеет n – строк и m – столбцов.

Матрица является квадратной, когда количество столбцов и строк равно между собой. Матрицей – вектором называется матрица из одного столбца с бесконечно возможным количеством строк. Матрица с единицами по одной из диагоналей и прочими нулевыми элементами называется единичной.

Обратная матрица – это такая матрица при умножении на которую исходная превращается в единичную, такая матрица существует только для исходной квадратной.

Правила преобразования системы уравнений в матрицу

Применительно к системам уравнений в качестве чисел матрицы записывают коэффициенты и свободные члены уравнений, одно уравнение – одна строка матрицы.

Строка матрицы называется ненулевой, если хотя бы один элемент строки не равен нулю. Поэтому если в каком-либо из уравнений количество переменных разнится, то необходимо на месте отсутствующей неизвестной вписать нуль.

Столбцы матрицы должны строго соответствовать переменным. Это означает что коэффициенты переменной x могут быть записаны только в один столбец, например первый, коэффициент неизвестной y – только во второй.

При умножении матрицы все элементы матрицы последовательно умножаются на число.

Варианты нахождения обратной матрицы

Формула нахождения обратной матрицы довольно проста: K -1 = 1 / |K|, где K -1 – обратная матрица, а |K| – определитель матрицы. |K| не должен быть равен нулю, тогда система имеет решение.

Определитель легко вычисляется для матрицы “два на два”, необходимо лишь помножить друг на друга элементы по диагонали. Для варианта “три на три” существует формула |K|=a 1 b 2 c 3 + a 1 b 3 c 2 + a 3 b 1 c 2 + a 2 b 3 c 1 + a 2 b 1 c 3 + a 3 b 2 c 1 . Можно воспользоваться формулой, а можно запомнить что необходимо взять по одному элементу из каждой строки и каждого столбца так, чтобы в произведении не повторялись номера столбцов и строк элементов.

Решение примеров систем линейных уравнений матричным методом

Матричный способ поиска решения позволяет сократить громоздкие записи при решении систем с большим количеством переменных и уравнений.

В примере a nm – коэффициенты уравнений, матрица – вектор x n – переменные, а b n – свободные члены.

Решение систем методом Гаусса

В высшей математике способ Гаусса изучают совместно с методом Крамера, а процесс поиска решения систем так и называется метод решения Гаусса – Крамера. Данные способы используют при нахождении переменных систем с большим количеством линейных уравнений.

Метод Гаусса очень похож на решения с помощью подстановок и алгебраического сложения, но более систематичен. В школьном курсе решение способом Гаусса применяется для систем из 3 и 4 уравнений. Цель метода состоит в приведении системы к виду перевернутой трапеции. Путем алгебраических преобразований и подстановок находится значение одной переменной в одном из уравнении системы. Второе уравнение представляет собой выражение с 2-мя неизвестными, ну а 3 и 4 – соответственно с 3-мя и 4-мя переменными.

После приведения системы к описанному виду, дальнейшее решение сводится к последовательной подстановке известных переменных в уравнения системы.

В школьных учебниках для 7 класса пример решения методом Гаусса описан следующим образом:

Как видно из примера, на шаге (3) было получено два уравнения 3x 3 -2x 4 =11 и 3x 3 +2x 4 =7. Решение любого из уравнений позволит узнать одну из переменных x n .

Теорема 5, о которой упоминается в тексте, гласит что если одно из уравнений системы заменить равносильным, то полученная система будет также равносильна исходной.

Метод Гаусса труден для восприятия учеников средней школы, но является одним из наиболее интересных способов для развития смекалки детей, обучающихся по программе углубленного изучения в математических и физических классах.

Для простоты записи вычислений принято делать следующим образом:

Коэффициенты уравнений и свободные члены записываются в виде матрицы, где каждая строка матрицы соотносится с одним из уравнений системы. отделяет левую часть уравнения от правой. Римскими цифрами обозначаются номера уравнений в системе.

Сначала записывают матрицу, с которой предстоит работать, затем все действия проводимые с одной из строк. Полученную матрицу записывают после знака “стрелка” и продолжают выполнять необходимые алгебраические действия до достижения результата.

В итоге должна получиться матрица в которой по одной из диагоналей стоят 1, а все другие коэффициенты равны нулю, то есть матрицу приводят к единичному виду. Нельзя забывать производить вычисления с цифрами обеих частей уравнения.

Данный способ записи менее громоздкий и позволяет не отвлекаться на перечисление многочисленных неизвестных.

Свободное применение любого способа решения потребует внимательности и определенного опыта. Не все методы имеют прикладной характер. Какие-то способы поиска решений более предпочтительны в той иной области деятельности людей, а другие существуют в целях обучения.

Системы линейных уравнений (Лекция №14)

Системой m линейных уравнений с n неизвестными называется система вида

где aij и bi (i=1,…,m; b=1,…,n) – некоторые известные числа, а x1,…,xn – неизвестные. В обозначении коэффициентов aij первый индекс iобозначает номер уравнения, а второй j – номер неизвестного, при котором стоит этот коэффициент.

Коэффициенты при неизвестных будем записывать в виде матрицы , которую назовём матрицей системы.

Числа, стоящие в правых частях уравнений, b1,…,bm называются свободными членами.

Совокупность n чисел c1,…,cn называется решением данной системы, если каждое уравнение системы обращается в равенство после подстановки в него чисел c1,…,cn вместо соответствующих неизвестных x1,…,xn.

Наша задача будет заключаться в нахождении решений системы. При этом могут возникнуть три ситуации:

  1. Система может иметь единственное решение.
  2. Система может иметь бесконечное множество решений. Например, . Решением этой системы является любая пара чисел, отличающихся знаком.
  3. И третий случай, когда система вообще не имеет решения. Например, , если бы решение существовало, то x1 + x2 равнялось бы одновременно нулю и единице.

Система линейных уравнений, имеющая хотя бы одно решение, называется совместной. В противном случае, т.е. если система не имеет решений, то она называется несовместной.

Рассмотрим способы нахождения решений системы.

МАТРИЧНЫЙ МЕТОД РЕШЕНИЯ СИСТЕМ ЛИНЕЙНЫХ УРАВНЕНИЙ

Матрицы дают возможность кратко записать систему линейных уравнений. Пусть дана система из 3-х уравнений с тремя неизвестными:

Рассмотрим матрицу системы и матрицы столбцы неизвестных и свободных членов

Найдем произведение

т.е. в результате произведения мы получаем левые части уравнений данной системы. Тогда пользуясь определением равенства матриц данную систему можно записать в виде

или короче AX=B.

Здесь матрицы A и B известны, а матрица X неизвестна. Её и нужно найти, т.к. её элементы являются решением данной системы. Это уравнение называют матричным уравнением.

Пусть определитель матрицы отличен от нуля |A| ≠ 0. Тогда матричное уравнение решается следующим образом. Умножим обе части уравнения слева на матрицу A-1, обратную матрице A: . Поскольку A-1A = E и EX = X, то получаем решение матричного уравнения в виде X = A-1B.

Заметим, что поскольку обратную матрицу можно найти только для квадратных матриц, то матричным методом можно решать только те системы, в которых число уравнений совпадает с числом неизвестных. Однако, матричная запись системы возможна и в случае, когда число уравнений не равно числу неизвестных, тогда матрица A не будет квадратной и поэтому нельзя найти решение системы в виде X = A-1B.

Примеры. Решить системы уравнений.

  1. Найдем матрицу обратную матрице A.

    ,

    Таким образом, x = 3, y = – 1.

  2. Итак, х1=4,х2=3,х3=5.

  3. Решите матричное уравнение: XA+B=C, где

    Выразим искомую матрицу X из заданного уравнения.

    Найдем матрицу А-1.

    Проверка:

  4. Решите матричное уравнение AX+B=C, где

    Из уравнения получаем .

    Следовательно,

ПРАВИЛО КРАМЕРА

Рассмотрим систему 3-х линейных уравнений с тремя неизвестными:

Определитель третьего порядка, соответствующий матрице системы, т.е. составленный из коэффициентов при неизвестных,

называется определителем системы.

Составим ещё три определителя следующим образом: заменим в определителе D последовательно 1, 2 и 3 столбцы столбцом свободных членов

Тогда можно доказать следующий результат.

Теорема (правило Крамера). Если определитель системы Δ ≠ 0, то рассматриваемая система имеет одно и только одно решение, причём

Доказательство. Итак, рассмотрим систему 3-х уравнений с тремя неизвестными. Умножим 1-ое уравнение системы на алгебраическое дополнение A11 элемента a11, 2-ое уравнение – на A21 и 3-е – на A31:

Сложим эти уравнения:

Рассмотрим каждую из скобок и правую часть этого уравнения. По теореме о разложении определителя по элементам 1-го столбца

.

Далее рассмотрим коэффициенты при x2:

Аналогично можно показать, что и .

Наконец несложно заметить, что

Таким образом, получаем равенство: .

Следовательно, .

Аналогично выводятся равенства и , откуда и следует утверждение теоремы.

Таким образом, заметим, что если определитель системы Δ ≠ 0, то система имеет единственное решение и обратно. Если же определитель системы равен нулю, то система либо имеет бесконечное множество решений, либо не имеет решений, т.е. несовместна.

Примеры. Решить систему уравнений

  1. Итак, х=1, у=2, z=3.

  2. Решите систему уравнений при различных значениях параметра p:

    Система имеет единственное решение, если Δ ≠ 0.

    . Поэтому .

    1. При
    2. При p = 30 получаем систему уравнений которая не имеет решений.
    3. При p = –30 система принимает вид и, следовательно, имеет бесконечное множество решений x=y, yÎR.

МЕТОД ГАУССА

Ранее рассмотренные методы можно применять при решении только тех систем, в которых число уравнений совпадает с числом неизвестных, причём определитель системы должен быть отличен от нуля. Метод Гаусса является более универсальным и пригоден для систем с любым числом уравнений. Он заключается в последовательном исключении неизвестных из уравнений системы.

Вновь рассмотрим систему из трёх уравнений с тремя неизвестными:

.

Первое уравнение оставим без изменения, а из 2-го и 3-го исключим слагаемые, содержащие x1. Для этого второе уравнение разделим на а21 и умножим на –а11, а затем сложим с 1-ым уравнением. Аналогично третье уравнение разделим на а31 и умножим на –а11, а затем сложим с первым. В результате исходная система примет вид:

Теперь из последнего уравнения исключим слагаемое, содержащее x2. Для этого третье уравнение разделим на , умножим на и сложим со вторым. Тогда будем иметь систему уравнений:

Отсюда из последнего уравнения легко найти x3, затем из 2-го уравнения x2 и, наконец, из 1-го – x1.

При использовании метода Гаусса уравнения при необходимости можно менять местами.

Часто вместо того, чтобы писать новую систему уравнений, ограничиваются тем, что выписывают расширенную матрицу системы:

и затем приводят её к треугольному или диагональному виду с помощью элементарных преобразований.

К элементарным преобразованиям матрицы относятся следующие преобразования:

  1. перестановка строк или столбцов;
  2. умножение строки на число, отличное от нуля;
  3. прибавление к одной строке другие строки.

Примеры: Решить системы уравнений методом Гаусса.

  1. Вернувшись к системе уравнений, будем иметь

  2. Выпишем расширенную матрицу системы и сведем ее к треугольному виду.

    Вернувшись к системе уравнений, несложно заметить, что третье уравнения системы будет ложным, а значит, система решений не имеет.

  3. Разделим вторую строку матрицы на 2 и поменяем местами первый и третий столбики. Тогда первый столбец будет соответствовать коэффициентам при неизвестной z, а третий – при x.

    Вернемся к системе уравнений.

    Из третьего уравнения выразим одну неизвестную через другую и подставим в первое.

Таким образом, система имеет бесконечное множество решений.

Конспект урока по математике для групп СПО на тему “Системы линейных алгебраических уравнений”

Конспект урока

Исследование системы линейных алгебраических уравнений

  Системой линейных алгебраических уравнений (СЛАУ) называется система вида:

Если  , то система линейных алгебраических уравнений называется однородной, в противном случае – неоднородной.

Совокупность значения неизвестных переменных  , при которых все уравнения системы обращаются в тождества, называется решением СЛАУ.

Если существует хотя бы одно решение системы линейных алгебраических уравнений, то она называется совместной, в противном случае – несовместной.

Если СЛАУ имеет единственное решение, то она называется  определенной. Если решений больше одного, то система называется  неопределенной.

Эта система в матричной форме записи имеет вид ,где

Расширенной матрицей системы называется матрица , получаемая из матрицы А добавлением в качестве (n+1)-ого столбца столбца свободных членов (столбец свободных членов отделяют вертикальной линией от остальных столбцов).

Теорема Кронекера-Капелли

СЛАУ совместна тогда и только тогда, когда ранг матрицы системы равен рангу расширенной матрицы.

Следствия из теоремы Кронекера-Капелли

  1. Если ранг матрицы системы А не равен рангу расширенной матрицы , то СЛАУ несовместна (не имеет решений).

  2. Если , то СЛАУ является неопределённой (имеет бесконечное количество решений).

  3. Если , то СЛАУ является определённой (имеет ровно одно решение).

Пример

Исследовать систему уравнений

Решение:

Расширенная матрица системы

Прибавим к элементам 2-й строки элементы 3-й строки:

Разделим все элементы 2-й строки на 3:

Вычтем из элементов 2-й строки элементы 1-й строки:

т.е.; система неопределенная и имеет бесконечное множество решений.

Методы решения СЛАУ

Метод Гаусса

Метод Гаусса – универсальный метод для нахождения решения CЛАУ с любым количеством уравнений и неизвестных.

Метод Гаусса включает в себя прямой (приведение расширенной матрицы к треугольному виду) и обратный (получение нулей над главной диагональю расширенной матрицы) ходы.

Прямой ход называется методом Гаусса, обратный – методом Гаусса-Жордана, который отличается от первого только последовательностью исключения переменных.

Пример

Решить СЛАУ  методом Гаусса.

Решение:

Выпишем расширенную матрицу системы и поменяем первую и вторую строку

Отнимем от второй строки первую, умноженную на 2, от третьей – первую, умноженную на 3:

Все элементы третьей строки делим на два:

Поменяем местами вторую и третью строки:

Отнимаем от третьей строки вторую, умноженную на 3:

Разделим третью строку на 2:

Проведем обратный ход метода Гаусса (метод Гаусса-Жордана), то есть сделаем нули над главной диагональю.

Отнимем от второй строки третью:

Прибавим к первой строке вторую:

Тогда  

Метод Крамера

Метод Крамера — способ решения квадратных СЛАУ с ненулевым определителем матрицы.

Теорема Крамера.

Если определитель матрицы квадратной системы не равен нулю, то система совместна и имеет единственное решение, которое находится по формулам Крамера:

где  – определитель матрицы системы, 

 – определитель матрицы системы, где вместо  -го столбца стоит столбец правых частей.

Примеры

  1. Решить СЛАУ   методом Крамера.

Решение:

 Определитель матрицы системы:

Так как  , то по теореме Крамера система совместна и имеет единственное решение.

Ответ:

  1. Решить СЛАУ   методом Крамера.

Решение:

Определитель матрицы системы:

Так как определитель матрицы системы неравен нулю, то по теореме Крамера система совместна и имеет единственное решение.

Ответ:         

Выполнить задания:

  1. Исследовать систему уравнений.

  1. Решить систему методом Гаусса

  1. Решить систему методом Крамера:

Литература

1. Григорьев В.П.Дубинский Ю.А.Элементы высшей математики (учебник для студ. учреждений СПО) – М.,2014

2. Афанасьева О.Н., Бродский Я.С. Математика для техникумов. – Москва: Физматлит, 2005.

  1. Данко П.Е., Попов А.Г., Кожевникова Т.Я. Высшая математика в упражнениях и задачах. – Москва: Оникс, 2008.

  2. Подольский В.А. и др. Сборник задач по математике для техникумов. – М.: Высшая школа, 2005.

  3. Соловейчик И.Л., Лисичкин В.Т. Сборник задач по математике для техникумов. – Москва: Оникс 21 век, 2003.

Интернет- ресурсы:

1.http://de.ifmo.ruЭлектронный учебник.

2.http://www.exponenta.ru – Образовательный математический сайт.

3.http://www.mathnet.ru – Общероссийский математический портал Math-Net.Ru

4. https://ru.onlinemschool.com/ – Общероссийский математический портал OnlineMSchool 

Решение системных линейных уравнений методом крамера. Правило Крамера

Методы Крамера и Гаусса – одни из самых популярных методов решения СЛАУ . К тому же, в ряде случаев целесообразно использовать именно конкретные методы. Сессия близка, и сейчас самое время повторить или освоить их с нуля. Сегодня разбираемся с решением методом Крамера. Ведь решение системы линейных уравнений методом Крамера – весьма полезный навык.

Системы линейных алгебраических уравнений

Система линейных алгебраических уравнений – система уравнений вида:

Набор значений x , при котором уравнения системы обращаются в тождества, называется решением системы, a и b – вещественные коэффициенты. Простенькую систему, состоящую из двух уравнений с двумя неизвестными, можно решить в уме либо выразив одну переменную через другую. Но переменных (иксов) в СЛАУ может быть гораздо больше двух, и здесь простыми школьными манипуляциями не обойтись. Что же делать? Например, решать СЛАУ методом Крамера!

Итак, пусть система состоит из n уравнений с n неизвестными.

Такую систему можно переписать в матричном виде

Здесь A – основная матрица системы, X и B , соответственно, матрицы-столбцы неизвестных переменных и свободных членов.

Решение СЛАУ методом Крамера

Если определитель главной матрицы не равен нулю (матрица невырожденная), систему можно решать по методу Крамера.

Согласно методу Крамера, решение находится по формулам:

Здесь дельта – определитель главной матрицы, а дельта x n-ное – определитель, полученный из определителя главной матрицы путем заменой n-ного столбца на столбец свободных членов.

В этом и заключается вся суть метода Крамера. Подставляя найденные по вышеприведенным формулам значения x в искомую систему, убеждаемся в правильности (или наоборот) нашего решения. Чтобы Вы быстрее уловили суть, приведем ниже пример подробного решения СЛАУ методом Крамера:

Даже если у Вас не получится с первого раза, не расстраивайтесь! Немного практики, и Вы начнете щелкать СЛАУ как орешки. Более того, сейчас совершенно необязательно корпеть над тетрадью, решая громоздкие выкладки и исписывая стержень. Можно легко решить СЛАУ методом Крамера в режиме онлайн, лишь подставив в готовую форму коэффициенты. Испробовать онлайн калькулятор решения методом Крамера можно, к примеру, на этом сайте .

А если система оказалась упорной и не сдается, Вы всегда можете обратиться за помощью к нашим авторам, например, чтобы . Будь в системе хоть 100 неизвестных, мы обязательно решим ее верно и точно в срок!


2. Решение систем уравнений матричным методом (при помощи обратной матрицы).
3. Метод Гаусса решения систем уравнений.

Метод Крамера.

Метод Крамера применяется для решения систем линейных алгебраических уравнений (СЛАУ ).

Формулы на примере системы из двух уравнений с двумя переменными.
Дано: Решить методом Крамера систему

Относительно переменных х и у .
Решение:
Найдем определитель матрицы, составленный из коэффициентов системы Вычисление определителей. :



Применим формулы Крамера и найдем значения переменных:
и .
Пример 1:
Решить систему уравнений:

относительно переменных х и у .
Решение:


Заменим в этом определителе первый столбец столбцом коэффициентов из правой части системы и найдем его значение:

Сделаем аналогичное действие, заменив в первом определителе второй столбец:

Применим формулы Крамера и найдем значения переменных:
и .
Ответ:
Замечание: Этим методом можно решать системы и большей размерности.

Замечание: Если получается, что , а делить на ноль нельзя, то говорят, что система не имеет единственного решения. В этом случае система имеет или бесконечно много решений или не имеет решений вообще.

Пример 2 (бесконечное количество решений):

Решить систему уравнений:

относительно переменных х и у .
Решение:
Найдем определитель матрицы, составленный из коэффициентов системы:

Решение систем методом подстановки.

Первое из уравнений системы — равенство, верное при любых значениях переменных (потому что 4 всегда равно 4). Значит, остается только одно уравнение. Это уравнение связи между переменными .
Получили, решением системы являются любые пары значений переменных, связанных между собой равенством .
Общее решение запишется так:
Частные решения можно определять выбирая произвольное значение у и вычисляя х по этому равенству связи.

и т.д.
Таких решений бесконечно много.
Ответ: общее решение
Частные решения:

Пример 3 (решений нет, система несовместна):

Решить систему уравнений:

Решение:
Найдем определитель матрицы, составленный из коэффициентов системы:

Применять формулы Крамера нельзя. Решим эту систему методом подстановки

Второе уравнение системы — равенство, неверное ни при каких значениях переменных (конечно же, так как -15 не равно 2). Если одно из уравнений системы не верно ни при каких значениях переменных, то и вся системы не имеет решений.
Ответ: решений нет

Метод Крамера основан на использовании определителей в решении систем линейных уравнений. Это значительно ускоряет процесс решения.

Метод Крамера может быть использован в решении системы стольких линейных уравнений, сколько в каждом уравнении неизвестных. Если определитель системы не равен нулю, то метод Крамера может быть использован в решении, если же равен нулю, то не может. Кроме того, метод Крамера может быть использован в решении систем линейных уравнений, имеющих единственное решение.

Определение . Определитель, составленный из коэффициентов при неизвестных, называется определителем системы и обозначается (дельта).

Определители

получаются путём замены коэффициентов при соответствующих неизвестных свободными членами:

;

.

Теорема Крамера . Если определитель системы отличен от нуля, то система линейных уравнений имеет одно единственное решение, причём неизвестное равно отношению определителей. В знаменателе – определитель системы, а в числителе – определитель, полученный из определителя системы путём замены коэффициентов при этом неизвестном свободными членами. Эта теорема имеет место для системы линейных уравнений любого порядка.

Пример 1. Решить систему линейных уравнений:

Согласно теореме Крамера имеем:

Итак, решение системы (2):

онлайн-калькулятором , решающим методом Крамера.

Три случая при решении систем линейных уравнений

Как явствует из теоремы Крамера , при решении системы линейных уравнений могут встретиться три случая:

Первый случай: система линейных уравнений имеет единственное решение

(система совместна и определённа)

Второй случай: система линейных уравнений имеет бесчисленное множество решений

(система совместна и неопределённа)

** ,

т.е. коэффициенты при неизвестных и свободные члены пропорциональны.

Третий случай: система линейных уравнений решений не имеет

(система несовместна)

Итак, система m линейных уравнений с n переменными называется несовместной , если у неё нет ни одного решения, и совместной , если она имеет хотя бы одно решение. Совместная система уравнений, имеющая только одно решение, называется определённой , а более одного – неопределённой .

Примеры решения систем линейных уравнений методом Крамера

Пусть дана система

.

На основании теоремы Крамера

………….
,

где

определитель системы. Остальные определители получим, заменяя столбец с коэффициентами соответствующей переменной (неизвестного) свободными членами:

Пример 2.

.

Следовательно, система является определённой. Для нахождения её решения вычисляем определители

По формулам Крамера находим:

Итак, (1; 0; -1) – единственное решение системы.

Для проверки решений систем уравнений 3 Х 3 и 4 Х 4 можно воспользоваться онлайн-калькулятором , решающим методом Крамера.

Если в системе линейных уравнений в одном или нескольких уравнениях отсутствуют какие-либо переменные, то в определителе соответствующие им элементы равны нулю! Таков следующий пример.

Пример 3. Решить систему линейных уравнений методом Крамера:

.

Решение. Находим определитель системы:

Посмотрите внимательно на систему уравнений и на определитель системы и повторите ответ на вопрос, в каких случаях один или несколько элементов определителя равны нулю. Итак, определитель не равен нулю, следовательно, система является определённой. Для нахождения её решения вычисляем определители при неизвестных

По формулам Крамера находим:

Итак, решение системы – (2; -1; 1).

Для проверки решений систем уравнений 3 Х 3 и 4 Х 4 можно воспользоваться онлайн-калькулятором , решающим методом Крамера.

К началу страницы

Продолжаем решать системы методом Крамера вместе

Как уже говорилось, если определитель системы равен нулю, а определители при неизвестных не равны нулю, система несовместна, то есть решений не имеет. Проиллюстрируем следующим примером.

Пример 6. Решить систему линейных уравнений методом Крамера:

Решение. Находим определитель системы:

Определитель системы равен нулю, следовательно, система линейных уравнений либо несовместна и определённа, либо несовместна, то есть не имеет решений. Для уточнения вычисляем определители при неизвестных

Определители при неизвестных не равны нулю, следовательно, система несовместна, то есть не имеет решений.

Для проверки решений систем уравнений 3 Х 3 и 4 Х 4 можно воспользоваться онлайн-калькулятором , решающим методом Крамера.

В задачах на системы линейных уравнений встречаются и такие, где кроме букв, обозначающих переменные, есть ещё и другие буквы. Эти буквы обозначают некоторое число, чаще всего действительное. На практике к таким уравнениям и системам уравнений приводят задачи на поиск общих свойств каких-либо явлений и предметов. То есть, изобрели вы какой-либо новый материал или устройство, а для описания его свойств, общих независимо от величины или количества экземпляра, нужно решить систему линейных уравнений, где вместо некоторых коэффициентов при переменных – буквы. За примерами далеко ходить не надо.

Следующий пример – на аналогичную задачу, только увеличивается количество уравнений, переменных, и букв, обозначающих некоторое действительное число.

Пример 8. Решить систему линейных уравнений методом Крамера:

Решение. Находим определитель системы:

Находим определители при неизвестных

Пусть система линейных уравнений содержит столько уравнений, каково количество независимых переменных, т.е. имеет вид

Такие системы линейных уравнений называются квадратными. Определитель, составленный из коэффициентов при независимых переменных системы (1.5), называется главным определителем системы. Мы будем обозначать его греческой буквой D. Таким образом,

. (1.6)

Если в главном определителе произвольный (j -ый) столбец, заменить столбцом свободных членов системы (1.5), то можно получить еще n вспомогательных определителей:

(j = 1, 2, …, n ). (1.7)

Правило Крамера решения квадратных систем линейных уравнений заключается в следующем. Если главный определитель D системы (1.5) отличен от нуля, то система имеет и притом единственное решение, которое можно найти по формулам:

(1.8)

Пример 1.5. Методом Крамера решить систему уравнений

.

Вычислим главный определитель системы:

Так как D¹0, то система имеет единственное решение, которое можно найти по формулам (1.8):

Таким образом,

Действия над матрицами

1. Умножение матрицы на число. Операция умножения матрицы на число определяется следующим образом.

2. Для того чтобы умножить матрицу на число, нужно все ее элементы умножить на это число. То есть

. (1.9)

Пример 1.6. .

Сложение матриц.

Данная операция вводится только для матриц одного и того же порядка.

Для того чтобы сложить две матрицы, необходимо к элементам одной матрицы прибавить соответствующие элементы другой матрицы:

(1.10)
Операция сложения матриц обладает свойствами ассоциативности и коммутативности.

Пример 1.7. .

Умножение матриц.

Если число столбцов матрицы А совпадает с числом строк матрицы В , то для таких матриц вводится операция умножения:

2

Таким образом, при умножении матрицы А размерности m ´n на матрицу В размерности n ´k мы получаем матрицу С размерности m ´k . При этом элементы матрицы С вычисляются по следующим формулам:

Задача 1.8. Найти, если это возможно, произведение матриц AB и BA :

Решение. 1) Для того чтобы найти произведение AB , необходимо строки матрицы A умножить на столбцы матрицы B :

2) Произведение BA не существует, т. к. количество столбцов матрицы B не совпадает с количеством строк матрицы A .

Обратная матрица. Решение систем линейных уравнений матричным способом

Матрица A – 1 называется обратной к квадратной матрице А , если выполнено равенство:

где через I обозначается единичная матрица того же порядка, что и матрица А :

.

Для того чтобы квадратная матрица имела обратную необходимо и достаточно, чтобы ее определитель был отличен от нуля. Обратную матрицу находят по формуле:

, (1.13)

где A ij – алгебраические дополнения к элементам a ij матрицы А (заметим, что алгебраические дополнения к строкам матрицы А располагаются в обратной матрице в виде соответствующих столбцов).

Пример 1.9. Найти обратную матрицу A – 1 к матрице

.

Обратную матрицу найдем по формуле (1.13), которая для случая n = 3 имеет вид:

.

Найдем det A = | A | = 1 × 3 × 8 + 2 × 5 × 3 + 2 × 4 × 3 – 3 × 3 × 3 – 1 × 5 × 4 – 2 × 2 × 8 = 24 + 30 + 24 – 27 – 20 – 32 = – 1. Так как определитель исходной матрицы отличен от нуля, то обратная матрица существует.

1) Найдем алгебраические дополнения A ij :

Для удобства нахождения обратной матрицы, алгебраические дополнения к строкам исходной матрицы мы расположили в соответствующие столбцы.

Из полученных алгебраических дополнений составим новую матрицу и разделим ее на определитель det A . Таким образом, мы получим обратную матрицу:

Квадратные системы линейных уравнений с отличным от нуля главным определителем можно решать с помощью обратной матрицы. Для этого систему (1.5) записывают в матричном виде:

где

Умножая обе части равенства (1.14) слева на A – 1 , мы получим решение системы:

, откуда

Таким образом, для того чтобы найти решение квадратной системы, нужно найти обратную матрицу к основной матрице системы и умножить ее справа на матрицу-столбец свободных членов.

Задача 1.10. Решить систему линейных уравнений

с помощью обратной матрицы.

Решение. Запишем систему в матричном виде: ,

где – основная матрица системы, – столбец неизвестных и – столбец свободных членов. Так как главный определитель системы , то основная матрица системы А имеет обратную матрицу А -1 . Для нахождения обратной матрицы А -1 , вычислим алгебраические дополнения ко всем элементам матрицы А :

Из полученных чисел составим матрицу (причем алгебраические дополнения к строкам матрицы А запишем в соответствующие столбцы) и разделим ее на определитель D. Таким образом, мы нашли обратную матрицу:

Решение системы находим по формуле (1.15):

Таким образом,

Решение систем линейных уравнений методом обыкновенных жордановых исключений

Пусть дана произвольная (не обязательно квадратная) система линейных уравнений:

(1.16)

Требуется найти решение системы, т.е. такой набор переменных , который удовлетворяет всем равенствам системы (1.16). В общем случае система (1.16) может иметь не только одно решение, но и бесчисленное множество решений. Она может так же вообще не иметь решений.

При решении подобных задач используется хорошо известный из школьного курса метод исключения неизвестных, который еще называется методом обыкновенных жордановых исключений. Суть данного метода заключается в том, что в одном из уравнений системы (1.16) одна из переменных выражается через другие переменные. Затем эта переменная подставляется в другие уравнения системы. В результате получается система, содержащая на одно уравнение и на одну переменную меньше, чем исходная система. Уравнение, из которого выражалась переменная, запоминается.

Этот процесс повторяется до тех пор, пока в системе не останется одно последнее уравнение. В процессе исключения неизвестных некоторые уравнения могут превратиться в верные тождества, например . Такие уравнения из системы исключаются, так как они выполняются при любых значениях переменных и, следовательно, не оказывают влияния на решение системы. Если в процессе исключения неизвестных хотя бы одно уравнение становится равенством, которое не может выполняться ни при каких значениях переменных (например ), то мы делаем вывод, что система не имеет решения.

Если в ходе решения противоречивых уравнений не возникло, то из последнего уравнения находится одна из оставшихся в нем переменных. Если в последнем уравнении осталась только одна переменная, то она выражается числом. Если в последнем уравнении остаются еще и другие переменные, то они считаются параметрами, и выраженная через них переменная будет функцией этих параметров. Затем совершается так называемый «обратный ход». Найденную переменную подставляют в последнее запомненное уравнение и находят вторую переменную. Затем две найденные переменные подставляют в предпоследнее запомненное уравнение и находят третью переменную, и так далее, вплоть до первого запомненного уравнения.

В результате мы получаем решение системы. Данное решение будет являться единственным, если найденные переменные будут числами. Если же первая найденная переменная, а затем и все остальные будут зависеть от параметров, то система будет иметь бесчисленное множество решений (каждому набору параметров соответствует новое решение). Формулы, позволяющие найти решение системы в зависимости от того или иного набора параметров, называются общим решением системы.

Пример 1.11.

x

После запоминания первого уравнения и приведения подобных членов во втором и третьем уравнении мы приходим к системе:

Выразим y из второго уравнения и подставим его в первое уравнение:

Запомним второе уравнение, а из первого найдем z :

Совершая обратный ход, последовательно найдем y и z . Для этого сначала подставим в последнее запомненное уравнение , откуда найдем y :

.

Затем подставим и в первое запомненное уравнение , откуда найдем x :

Задача 1.12. Решить систему линейных уравнений методом исключения неизвестных:

. (1.17)

Решение. Выразим из первого уравнения переменную x и подставим ее во второе и третье уравнения:

.

Запомним первое уравнение

В данной системе первое и второе уравнения противоречат друг другу. Действительно, выражая y , получим, что 14 = 17. Данное равенство не выполняется, ни при каких значениях переменных x , y , и z . Следовательно, система (1.17) несовместна, т.е. не имеет решения.

Читателям предлагаем самостоятельно проверить, что главный определитель исходной системы (1.17) равен нулю.

Рассмотрим систему, отличающуюся от системы (1.17) всего лишь одним свободным членом.

Задача 1.13. Решить систему линейных уравнений методом исключения неизвестных:

. (1.18)

Решение. Как и прежде, выразим из первого уравнения переменную x и подставим ее во второе и третье уравнения:

.

Запомним первое уравнение и приведем подобные члены во втором и третьем уравнении. Мы приходим к системе:

Выражая y из первого уравнения и подставляя его во второе уравнение , мы получим тождество 14 = 14, которое не влияет на решение системы, и, следовательно, его можно из системы исключить.

В последнем запомненном равенстве переменную z будем считать параметром. Полагаем . Тогда

Подставим y и z в первое запомненное равенство и найдем x :

.

Таким образом, система (1.18) имеет бесчисленное множество решений, причем любое решение можно найти по формулам (1.19), выбирая произвольное значение параметра t :

(1.19)
Так решениями системы, например, являются следующие наборы переменных (1; 2; 0), (2; 26; 14) и т. д. Формулы (1.19) выражают общее (любое) решение системы (1.18).

В том случае, когда исходная система (1.16) имеет достаточно большое количество уравнений и неизвестных, указанный метод обыкновенных жордановых исключений представляется громоздким. Однако это не так. Достаточно вывести алгоритм пересчета коэффициентов системы при одном шаге в общем виде и оформить решение задачи в виде специальных жордановых таблиц.

Пусть дана система линейных форм (уравнений):

, (1.20)
где x j – независимые (искомые) переменные, a ij – постоянные коэффициенты
(i = 1, 2,…, m ; j = 1, 2,…, n ). Правые части системы y i (i = 1, 2,…, m ) могут быть как переменными (зависимыми), так и константами. Требуется найти решений данной системы методом исключения неизвестных.

Рассмотрим следующую операцию, называемую в дальнейшем «одним шагом обыкновенных жордановых исключений». Из произвольного (r -го) равенства выразим произвольную переменную (x s ) и подставим во все остальные равенства. Разумеется, это возможно только в том случае, когда a rs ¹ 0. Коэффициент a rs называется разрешающим (иногда направляющим или главным) элементом.

Мы получим следующую систему:

. (1.21)

Из s -го равенства системы (1.21) мы впоследствии найдем переменную x s (после того, как будут найдены остальные переменные). S -я строка запоминается и в дальнейшем из системы исключается. Оставшаяся система будет содержать на одно уравнение и на одну независимую переменную меньше, чем исходная система.

Вычислим коэффициенты полученной системы (1.21) через коэффициенты исходной системы (1.20). Начнем с r -го уравнения, которое после выражения переменной x s через остальные переменные будет выглядеть следующим образом:

Таким образом, новые коэффициенты r -го уравнения вычисляются по следующим формулам:

(1.23)
Вычислим теперь новые коэффициенты b ij (i ¹ r ) произвольного уравнения. Для этого подставим выраженную в (1.22) переменную x s в i -е уравнение системы (1.20):

После приведения подобных членов, получим:

(1.24)
Из равенства (1.24) получим формулы, по которым вычисляются остальные коэффициенты системы (1.21) (за исключением r -го уравнения):

(1.25)
Преобразование систем линейных уравнений методом обыкновенных жордановых исключений оформляется в виде таблиц (матриц). Эти таблицы получили название «жордановых».

Так, задаче (1.20) ставится в соответствие следующая жорданова таблица:

Таблица 1.1

x 1x 2x j x s x n
y 1 =a 11a 12a 1j a 1s a 1n
…………………………………………………………………..
y i =a i 1a i 2a ij a is a in
…………………………………………………………………..
y r =a r 1a r 2a rj a rsa rn
………………………………………………………………….
y n =a m 1a m 2a mj a ms a mn

Жорданова таблица 1.1 содержит левый заглавный столбец, в который записывают правые части системы (1.20) и верхнюю заглавную строку, в которую записывают независимые переменные.

Остальные элементы таблицы образуют основную матрицу коэффициентов системы (1.20). Если умножить матрицу А на матрицу , состоящую из элементов верхней заглавной строки, то получится матрица , состоящая из элементов левого заглавного столбца. То есть, по существу, жорданова таблица это матричная форма записи системы линейных уравнений: . Системе (1.21) при этом соответствует следующая жорданова таблица:

Таблица 1.2

x 1x 2x j y r x n
y 1 =b 11b 12b 1 j b 1 s b 1 n
…………………………………………………………………..
y i = b i 1b i 2b ij b is b in
…………………………………………………………………..
x s = b r 1b r 2b rj b rs b rn
………………………………………………………………….
y n = b m 1b m 2b mj b ms b mn

Разрешающий элемент a rs мы будем выделять жирным шрифтом. Напомним, что для осуществления одного шага жордановых исключений разрешающий элемент должен быть отличен от нуля. Строку таблицы, содержащую разрешающий элемент, называют разрешающей строкой. Столбец, содержащий разрешающий элемент, называют разрешающим столбцом. При переходе от данной таблицы к следующей таблице одна переменная (x s ) из верней заглавной строки таблицы перемещается в левый заглавный столбец и, наоборот, один из свободных членов системы (y r ) из левого заглавного столбца таблицы перемещается в верхнюю заглавную строку.

Опишем алгоритм пересчета коэффициентов при переходе от жордановой таблицы (1.1) к таблице (1.2), вытекающий из формул (1.23) и (1.25).

1. Разрешающий элемент заменяется обратным числом:

2. Остальные элементы разрешающей строки делятся на разрешающий элемент и изменяют знак на противоположный:

3. Остальные элементы разрешающего столбца делятся на разрешающий элемент:

4. Элементы, не попавшие в разрешающую строку и разрешающий столбец, пересчитываются по формулам:

Последняя формула легко запоминается, если заметить, что элементы, составляющие дробь , находятся на пересечении i -ой и r -ой строк и j -го и s -го столбцов (разрешающей строки, разрешающего столбца и той строки и столбца, на пересечении которых находится пересчитываемый элемент). Точнее, при запоминании формулы можно использовать следующую диаграмму:

-21-26-13-37

Совершая первый шаг жордановых исключений, в качестве разрешающего элемента можно выбрать любой элемент таблицы 1.3, расположенный в столбцах x 1 ,…, x 5 (все указанные элементы не равны нулю). Не следует только выбирать разрешающий элемент в последнем столбце, т.к. требуется находить независимые переменные x 1 ,…, x 5 . Выбираем, например, коэффициент 1 при переменной x 3 в третьей строке таблицы 1.3 (разрешающий элемент показан жирным шрифтом). При переходе к таблице 1.4 переменная x 3 из верхней заглавной строки меняется местами с константой 0 левого заглавного столбца (третья строка). При этом переменная x 3 выражается через остальные переменные.

Строку x 3 (табл.1.4) можно, предварительно запомнив, исключить из таблицы 1.4. Из таблицы 1.4 исключается так же третий столбец с нулем в верхней заглавной строке. Дело в том, что независимо от коэффициентов данного столбца b i 3 все соответствующие ему слагаемые каждого уравнения 0·b i 3 системы будут равны нулю. Поэтому указанные коэффициенты можно не вычислять. Исключив одну переменную x 3 и запомнив одно из уравнений, мы приходим к системе, соответствующей таблице 1.4 (с вычеркнутой строкой x 3). Выбирая в таблице 1.4 в качестве разрешающего элемента b 14 = -5, переходим к таблице 1.5. В таблице 1.5 запоминаем первую строку и исключаем ее из таблицы вместе с четвертым столбцом (с нулем наверху).

Таблица 1.5 Таблица 1.6

Из последней таблицы 1.7 находим: x 1 = – 3 + 2x 5 .

Последовательно подставляя уже найденные переменные в запомненные строки, находим остальные переменные:

Таким образом, система имеет бесчисленное множество решений. Переменной x 5 , можно придавать произвольные значения. Данная переменная выступает в роли параметра x 5 = t. Мы доказали совместность системы и нашли ее общее решение:

x 1 = – 3 + 2t

x 2 = – 1 – 3t

x 3 = – 2 + 4t . (1.27)
x 4 = 4 + 5t

x 5 = t

Придавая параметру t различные значения, мы получим бесчисленное множество решений исходной системы. Так, например, решением системы является следующий набор переменных (- 3; – 1; – 2; 4; 0).

В первой части мы рассмотрели немного теоретического материала, метод подстановки, а также метод почленного сложения уравнений системы. Всем, кто зашел на сайт через эту страницу рекомендую ознакомиться с первой частью. Возможно, некоторым посетителям покажется материал слишком простым, но по ходу решения систем линейных уравнений я сделал ряд очень важных замечаний и выводов, касающихся решения математических задач в целом.

А сейчас мы разберём правило Крамера, а также решение системы линейных уравнений с помощью обратной матрицы (матричный метод). Все материалы изложены просто, подробно и понятно, практически все читатели смогут научиться решать системы вышеуказанными способами.

Сначала мы подробно рассмотрим правило Крамера для системы двух линейных уравнений с двумя неизвестными. Зачем? – Ведь простейшую систему можно решить школьным методом, методом почленного сложения!

Дело в том, что пусть иногда, но встречается такое задание – решить систему двух линейных уравнений с двумя неизвестными по формулам Крамера. Во-вторых, более простой пример поможет понять, как использовать правило Крамера для более сложного случая – системы трех уравнений с тремя неизвестными.

Кроме того, существуют системы линейных уравнений с двумя переменными, которые целесообразно решать именно по правилу Крамера!

Рассмотрим систему уравнений

На первом шаге вычислим определитель , его называют главным определителем системы .

метод Гаусса .

Если , то система имеет единственное решение, и для нахождения корней мы должны вычислить еще два определителя:
и

На практике вышеуказанные определители также могут обозначаться латинской буквой .

Корни уравнения находим по формулам:
,

Пример 7

Решить систему линейных уравнений

Решение : Мы видим, что коэффициенты уравнения достаточно велики, в правой части присутствуют десятичные дроби с запятой. Запятая – довольно редкий гость в практических заданиях по математике, эту систему я взял из эконометрической задачи.

Как решить такую систему? Можно попытаться выразить одну переменную через другую, но в этом случае наверняка получатся страшные навороченные дроби, с которыми крайне неудобно работать, да и оформление решения будет выглядеть просто ужасно. Можно умножить второе уравнение на 6 и провести почленное вычитание, но и здесь возникнут те же самые дроби.

Что делать? В подобных случаях и приходят на помощь формулы Крамера.

;

;

Ответ : ,

Оба корня обладают бесконечными хвостами, и найдены приближенно, что вполне приемлемо (и даже обыденно) для задач эконометрики.

Комментарии здесь не нужны, поскольку задание решается по готовым формулам, однако, есть один нюанс. Когда используете данный метод, обязательным фрагментом оформления задания является следующий фрагмент: «, значит, система имеет единственное решение» . В противном случае рецензент может Вас наказать за неуважение к теореме Крамера.

Совсем не лишней будет проверка, которую удобно провести на калькуляторе: подставляем приближенные значения в левую часть каждого уравнения системы. В результате с небольшой погрешностью должны получиться числа, которые находятся в правых частях.

Пример 8

Ответ представить в обыкновенных неправильных дробях. Сделать проверку.

Это пример для самостоятельного решения (пример чистового оформления и ответ в конце урока).

Переходим к рассмотрению правила Крамера для системы трех уравнений с тремя неизвестными:

Находим главный определитель системы:

Если , то система имеет бесконечно много решений или несовместна (не имеет решений). В этом случае правило Крамера не поможет, нужно использовать метод Гаусса .

Если , то система имеет единственное решение и для нахождения корней мы должны вычислить еще три определителя:
, ,

И, наконец, ответ рассчитывается по формулам:

Как видите, случай «три на три» принципиально ничем не отличается от случая «два на два», столбец свободных членов последовательно «прогуливается» слева направо по столбцам главного определителя.

Пример 9

Решить систему по формулам Крамера.

Решение : Решим систему по формулам Крамера.

, значит, система имеет единственное решение.

Ответ : .

Собственно, здесь опять комментировать особо нечего, ввиду того, что решение проходит по готовым формулам. Но есть пара замечаний.

Бывает так, что в результате вычислений получаются «плохие» несократимые дроби, например: .
Я рекомендую следующий алгоритм «лечения». Если под рукой нет компьютера, поступаем так:

1) Возможно, допущена ошибка в вычислениях. Как только Вы столкнулись с «плохой» дробью, сразу необходимо проверить, правильно ли переписано условие . Если условие переписано без ошибок, то нужно пересчитать определители, используя разложение по другой строке (столбцу).

2) Если в результате проверки ошибок не выявлено, то вероятнее всего, допущена опечатка в условии задания. В этом случае спокойно и ВНИМАТЕЛЬНО прорешиваем задание до конца, а затем обязательно делаем проверку и оформляем ее на чистовике после решения. Конечно, проверка дробного ответа – занятие неприятное, но зато будет обезоруживающий аргумент для преподавателя, который ну очень любит ставить минус за всякую бяку вроде . Как управляться с дробями, подробно расписано в ответе для Примера 8.

Если под рукой есть компьютер, то для проверки используйте автоматизированную программу, которую можно бесплатно скачать в самом начале урока. Кстати, выгоднее всего сразу воспользоваться программой (еще до начала решения), Вы сразу будете видеть промежуточный шаг, на котором допустили ошибку! Этот же калькулятор автоматически рассчитывает решение системы матричным методом.

Замечание второе. Время от времени встречаются системы в уравнениях которых отсутствуют некоторые переменные, например:

Здесь в первом уравнении отсутствует переменная , во втором – переменная . В таких случаях очень важно правильно и ВНИМАТЕЛЬНО записать главный определитель:
– на месте отсутствующих переменных ставятся нули.
Кстати определители с нулями рационально раскрывать по той строке (столбцу), в которой находится ноль, так как вычислений получается заметно меньше.

Пример 10

Решить систему по формулам Крамера.

Это пример для самостоятельного решения (образец чистового оформления и ответ в конце урока).

Для случая системы 4 уравнений с 4 неизвестными формулы Крамера записываются по аналогичным принципам. Живой пример можно посмотреть на уроке Свойства определителя. Понижение порядка определителя – пять определителей 4-го порядка вполне решабельны. Хотя задача уже весьма напоминает ботинок профессора на груди у студента-счастливчика.

Решение системы с помощью обратной матрицы

Метод обратной матрицы – это, по существу, частный случай матричного уравнения (см. Пример №3 указанного урока).

Для изучения данного параграфа необходимо уметь раскрывать определители, находить обратную матрицу и выполнять матричное умножение. Соответствующие ссылки будут даны по ходу объяснений.

Пример 11

Решить систему с матричным методом

Решение : Запишем систему в матричной форме:
, где

Пожалуйста, посмотрите на систему уравнений и на матрицы. По какому принципу записываем элементы в матрицы, думаю, всем понятно. Единственный комментарий: если бы в уравнениях отсутствовали некоторые переменные, то на соответствующих местах в матрице нужно было бы поставить нули.

Обратную матрицу найдем по формуле:
, где – транспонированная матрица алгебраических дополнений соответствующих элементов матрицы .

Сначала разбираемся с определителем:

Здесь определитель раскрыт по первой строке.

Внимание! Если , то обратной матрицы не существует, и решить систему матричным методом невозможно. В этом случае система решается методом исключения неизвестных (методом Гаусса) .

Теперь нужно вычислить 9 миноров и записать их в матрицу миноров

Справка: Полезно знать смысл двойных подстрочных индексов в линейной алгебре. Первая цифра – это номер строки, в которой находится данный элемент. Вторая цифра – это номер столбца, в котором находится данный элемент:

То есть, двойной подстрочный индекс указывает, что элемент находится в первой строке, третьем столбце, а, например, элемент находится в 3 строке, 2 столбце

Калькулятор Системы линейных уравнений

Этот калькулятор решает систему линейных уравнений любого типа с указанными шагами, используя метод исключения Гаусса-Жордана, метод обратной матрицы или правило Крамера.

Связанный калькулятор: Калькулятор системы уравнений

Через запятую, например, x + 2y = 5,3x + 5y = 14.

Оставьте поле пустым для автоматического определения или укажите такие переменные, как x, y (через запятую).

Метод:

Метод исключения Гаусса-Жордана Метод обратной матрицы Правило Крамера

Если калькулятор что-то не вычислил, или вы определили ошибку, или у вас предложение / отзыв, напишите в комментариях ниже.

Ваш ввод

Решите $$$ \ begin {case} 5 x – 2 y = 1 \\ x + 3 y = 7 \ end {cases} $$$ для $$$ x $$$, $ $$ y $$$ с использованием метода исключения Гаусса-Джордана.

Решение

Запишите расширенную матрицу: $$$ \ left [\ begin {array} {cc | c} 5 & -2 & 1 \\ 1 & 3 & 7 \ end {array} \ right] $ $$.

Выполните исключение Гаусса-Жордана (шаги см. В калькуляторе исключения Гаусса-Жордана): $$$ \ left [\ begin {array} {cc | c} 5 & -2 & 1 \\ 0 & \ frac {17 } {5} & \ frac {34} {5} \ end {array} \ right] $$$.

Обратный заменитель:

$$$ y = \ frac {\ frac {34} {5}} {\ frac {17} {5}} = 2 $$$

$$$ x = \ frac { 1 – \ left (-2 \ right) \ left (2 \ right)} {5} = 1 $$$

Ответ

$$$ x = 1 $$$, $$$ y = 2 $$$ A

3×3 Решатель Системы Уравнений

О правиле Крамера

Этот калькулятор использует правило Крамера для решения систем трех уравнений с тремя неизвестные.Правило Крамера можно сформулировать следующим образом:

Учитывая систему:

$$ \ begin {выровнено} a_1x + b_1y + c_1z = d_1 \\ а_2x + b_2y + c_2z = d_2 \\ a_3x + b_3y + c_3z = d_3 \ end {выровнен} $$

с

$$ D = \ left | \ begin {array} {ccc} a_1 и b_1 и c_1 \\ a_2 и b_2 и c_2 \\ a_3 & b_3 & c_3 \\ \ end {array} \ right | \ ne 0 $$ $$ D_x = \ left | \ begin {array} {ccc} d_1 & b_1 & c_1 \\ d_2 & b_2 & c_2 \\ d_3 & b_3 & c_3 \\ \ end {array} \ right | $$ $$ D_y = \ left | \ begin {array} {ccc} a_1 и d_1 и c_1 \\ a_2 & d_2 & c_2 \\ a_3 и d_3 и c_3 \\ \ end {array} \ right | $$ $$ D_z = \ left | \ begin {array} {ccc} a_1 и b_1 и d_1 \\ а_2 и b_2 и d_2 \\ a_3 & b_3 & d_3 \\ \ end {array} \ right | $$

, то решение этой системы:

$$ x = \ frac {D_x} {D} $$ $$ y = \ frac {D_y} {D} $$ $$ z = \ frac {D_z} {D} $$

Пример: Решите систему уравнений, используя правило Крамера

$$ \ begin {выровнено} 4x + 5y -2z = & -14 \\ 7x – ~ y + 2z = & 42 \\ 3x + ~ y + 4z = & 28 \\ \ end {выровнен} $$

Решение: Сначала мы вычисляем $ D, ~ D_x, ~ D_y $ и $ D_z $.

$$ \ begin {выровнено} & D ~~ = \ left | \ begin {массив} {ccc} {\ color {blue} {4}} & {\ color {red} {~ 5}} & {\ color {green} {- 2}} \\ {\ color {blue} {7}} & {\ color {red} {- 1}} & {\ color {green} {~ 2}} \\ {\ color {blue} {3}} & {\ color {red} {~ 1}} & {\ color {green} {~ 4}} \ end {array} \ right | = -16 + 30-14-6-8-140 = -154 \\ & D_x = \ left | \ begin {массив} {ccc} -14 & {\ color {red} {~ 5}} & {\ color {green} {- 2}} \\ ~ 42 & {\ color {red} {- 1}} & {\ color {green} {~ 2}} \\ ~ 28 & {\ color {red} {1}} & {\ color {green} {~ 4}} \ end {array} \ right | = 56 + 280 – 84 – 56 + 28 – 840 = -616 \\ & D_y = \ left | \ begin {array} {ccc} {\ color {blue} {4}} & -14 & {\ color {green} {- 2}} \\ {\ color {blue} {7}} & ~ 42 & {\ color {green} {~ 2}} \\ {\ color {blue} {3}} & ~ 28 & {\ color {green} {~ 4}} \ end {array} \ right | = 672 – 84 – 392 + 252 – 224 + 392 = 616 \\ & D_Z = \ left | \ begin {array} {ccc} {\ color {blue} {4}} & {\ color {red} {~ 5}} & -14 \\ {\ color {blue} {7}} & {\ color {red} {- 1}} & ~ 42 \\ {\ color {blue} {3}} & {\ color {red} {~ 1}} & ~ 28 \ end {array} \ right | = -112 + 630 – 98 – 42 – 168 – 980 = -770 \\ \ end {выровнен} $$

Следовательно,

$$ \ begin {выровнено} & x = \ frac {D_x} {D} = \ frac {-616} {- 154} = 4 \\ & y = \ frac {D_y} {D} = \ frac {616} {- 154} = -4 \\ & z = \ frac {D_z} {D} = \ frac {-770} {- 154} = 5 \ end {выровнен} $$

Примечание: Вы можете проверить решение с помощью вышеуказанного калькулятора

3.3: Решение систем с исключением Гаусса-Джордана

Цели обучения

  • Напишите расширенную матрицу системы уравнений.
  • Напишите систему уравнений из расширенной матрицы.
  • Решите систему линейных уравнений с помощью матриц и графического калькулятора.
  • Решайте финансовые приложения с помощью матриц и графического калькулятора.

Необходимые навыки

Перед тем, как начать, пройдите предварительный тест.

Введите в калькулятор следующие матрицы и затем выполните указанные операции. Если операция не может быть проведена, укажите причину.

\ (A = \ begin {bmatrix} 5 & 1 & -2 \\ 2 & 6 & 7 \\ 4 & 1 & −5 \ end {bmatrix} \), \ (B = \ begin {bmatrix} 3 & -7 \\ 0 & 1 \\ 2 & −8 \ end {bmatrix} \), \ (C = \ begin {bmatrix} 9 & 4 \\ 6 & -5 \\ 7 & −1 \ end {bmatrix} \)

а. \ (А \ cdot B \)

г. \ (B \ cdot A \)

г. \ (4B-2C \)

г.\ (А + С \)

Нажмите здесь, чтобы проверить свой ответ

а. \ (\ begin {bmatrix} 11 & -18 \\ 20 & -64 \\ 2 & 13 \ end {bmatrix} \)

г. Не определено, поскольку количество столбцов в матрице \ (B \) не соответствует количеству строк в матрице \ (A \).

г. \ (\ begin {bmatrix} -6 & -36 \\ – 12 & 14 \\ – 6 & −30 \ end {bmatrix} \)

г. Не определено, поскольку размер матрицы \ (A \) не соответствует размерам матрицы \ (C \).{th} \) века, но он по-прежнему считается одним из самых плодовитых математиков в истории. Его вклад в математику и физику охватывает такие области, как алгебра, теория чисел, анализ, дифференциальная геометрия, астрономия и оптика. Его открытия в области теории матриц изменили способ работы математиков за последние два столетия.

Рисунок \ (\ PageIndex {1} \): немецкий математик Карл Фридрих Гаусс (1777–1855).

Ранее в этой главе мы исследовали методы решения систем уравнений.В этом разделе мы изучим другую технику решения систем, на этот раз с использованием матриц.

Расширенные матрицы

Матрица может служить средством представления и решения системы уравнений. Чтобы выразить систему в матричной форме, мы извлекаем коэффициенты переменных и констант, и они становятся элементами матрицы. Мы используем вертикальную линию, чтобы отделить записи коэффициентов от констант, по сути заменяя знаки равенства. Когда система написана в такой форме, мы называем ее расширенной матрицей .

Например, рассмотрим следующую систему уравнений \ (2 × 2 \).

\ [\ begin {align *} 3x + 4y & = 7 \\ 4x-2y & = 5 \ end {align *} \]

Мы можем записать эту систему в виде расширенной матрицы:

\ (\ left [\ begin {array} {cc | c} 3 & 4 & 7 \\ 4 & -2 & 5 \ end {array} \ right] \)

Мы также можем написать матрицу, содержащую только коэффициенты. Это называется матрицей коэффициентов .

\ (\ begin {bmatrix} 3 & 4 \\ 4 & −2 \ end {bmatrix} \)

Трехкратная система уравнений , например

\ [\ begin {align *} 3x-y-z & = 0 \\ x + y & = 5 \\ 2x-3z & = 2 \ end {align *} \]

имеет матрицу коэффициентов

\ (\ begin {bmatrix} 3 & −1 & −1 \\ 1 & 1 & 0 \\ 2 & 0 & −3 \ end {bmatrix} \)

и представлена ​​расширенной матрицей

\ (\ left [\ begin {array} {ccc | c} 3 & −1 & −1 & 0 \\ 1 & 1 & 0 & 5 \\ 2 & 0 & −3 & 2 \ end {array} \ right] \)

Обратите внимание, что матрица написана так, что переменные выстраиваются в свои собственные столбцы: \ (x \) – члены идут в первый столбец, \ (y \) – термины во втором столбце, и \ (z \) – термины в третьем столбце.Очень важно, чтобы каждое уравнение было записано в стандартной форме \ (ax + by + cz = d \), чтобы переменные совпадали. Когда в уравнении отсутствует член переменной, коэффициент равен \ (0 \).

Как: по системе уравнений написать расширенную матрицу

  1. Запишите коэффициенты членов \ (x \) – числами в первом столбце.
  2. Запишите коэффициенты членов \ (y \) в виде чисел во втором столбце.
  3. Если есть \ (z \) – члены, запишите коэффициенты в виде чисел в третьем столбце.
  4. Нарисуйте вертикальную линию и напишите константы справа от линии.

Пример \ (\ PageIndex {1} \): написание расширенной матрицы для системы уравнений

Напишите расширенную матрицу для данной системы уравнений.

\ [\ begin {align *} x + 2y-z & = 3 \\ 2x-y + 2z & = 6 \\ x-3y + 3z & = 4 \ end {align *} \]

Решение

Расширенная матрица отображает коэффициенты переменных и дополнительный столбец для констант.

\ (\ left [\ begin {array} {ccc | c} 1 & 2 & −1 & 3 \\ 2 & −1 & 2 & 6 \\ 1 & −3 & 3 & 4 \ end {array} \ right] \)

Упражнение \ (\ PageIndex {1} \)

Запишите расширенную матрицу данной системы уравнений.

\ [\ begin {align *} 4x-3y & = 11 \\ 3x + 2y & = 4 \ end {align *} \]

Ответ

\ (\ left [\ begin {array} {cc | c} 4 & −3 & 11 \\ 3 & 2 & 4 \ end {array} \ right] \)

Написание системы уравнений из расширенной матрицы

Мы можем использовать расширенные матрицы, чтобы помочь нам решать системы уравнений, потому что они упрощают операции, когда системы не обременены переменными.Однако важно понимать, как переключаться между форматами, чтобы поиск решений был более плавным и интуитивно понятным. Здесь мы будем использовать информацию в расширенной матрице, чтобы записать систему уравнений в стандартной форме.

Пример \ (\ PageIndex {2} \): Написание системы уравнений из расширенной матричной формы

Найдите систему уравнений из расширенной матрицы.

\ (\ left [\ begin {array} {ccc | c} 1 & −3 & −5 & -2 \\ 2 & −5 & −4 & 5 \\ – 3 & 5 & 4 & 6 \ end {array} \ right] \)

Решение

Когда столбцы представляют переменные \ (x \), \ (y \) и \ (z \),

\ [\ left [\ begin {array} {ccc | c} 1 & -3 & -5 & -2 \\ 2 & -5 & -4 & 5 \\ – 3 & 5 & 4 & 6 \ end {array} \ right] \ rightarrow \ begin {align *} x-3y-5z & = -2 \\ 2x-5y-4z & = 5 \\ -3x + 5y + 4z & = 6 \ end {align *} \]

Упражнение \ (\ PageIndex {2} \)

Напишите систему уравнений из расширенной матрицы.

\ (\ left [\ begin {array} {ccc | c} 1 & -1 & 1 & 5 \\ 2 & -1 & 3 & 1 \\ 0 & 1 & 1 & -9 \ end {array} \ right] \)

Ответ

\ (\ begin {align *} x-y + z & = 5 \\ 2x-y + 3z & = 1 \\ y + z & = -9 \ end {align *} \)

Уменьшенная форма рядка-эшелон

Чтобы решить систему уравнений, мы хотим преобразовать ее матрицу в уменьшенную форму строки , в которой есть единицы вниз по главной диагонали от верхнего левого угла к нижнему правому углу, а нули в каждое положение выше и ниже главной диагонали, как показано.

Уменьшенная форма строки-эшелона \ (\ begin {bmatrix} 1 & 0 & 0 \\ 0 & 1 & 0 \\ 0 & 0 & 1 \ end {bmatrix} \)

Следующие расширенные матрицы представлены в сокращенной форме строки-эшелона.

\ (\ left [\ begin {array} {cc | c} 1 & 0 & -2 \\ 0 & 1 & 5 \ end {array} \ right] \), \ (\ left [\ begin {array} {ccc | c} 1 & 0 & 0 & 4 \\ 0 & 1 & 0 & 3 \\ 0 & 0 & 1 & 2 \ end {array} \ right] \)

Следующие расширенные матрицы не являются сокращенными строками.

\ (\ left [\ begin {array} {cc | c} 2 & 4 & -6 \\ 4 & 0 & 7 \ end {array} \ right] \), \ (\ left [\ begin {array} {ccc | c} 0 & 2 & 3 & 3 \\ 1 & 5 & 0 & 2 \\ 0 & 0 & 1 & 0 \ end {array} \ right] \)

Пример \ (\ PageIndex {3} \): матрицы в сокращенной форме строки-эшелон

Запишите систему уравнений из каждой из матриц в приведенной строчно-эшелонированной форме сверху. В чем преимущество этой формы?

а. \ (\ left [\ begin {array} {cc | c} 1 & 0 & -2 \\ 0 & 1 & 5 \ end {array} \ right] \)

г.\ (\ left [\ begin {array} {ccc | c} 1 & 0 & 0 & 4 \\ 0 & 1 & 0 & 3 \\ 0 & 0 & 1 & 2 \ end {array} \ right] \)

Решение

а. \ (\ begin {align *} x = -2 \\ y = 5 \ end {align *} \)

г. \ (\ begin {align *} x = 4 \\ y = 3 \\ z = 2 \ end {align *} \)

Преимущество сокращенной формы «строка-эшелон» состоит в том, что решение системы уравнений приводится в правом столбце.

УСТРАНЕНИЕ ПО ГАУСС-ИОРДАНИИ

Метод исключения Гаусса-Жордана относится к стратегии, используемой для получения уменьшенной строковой формы матрицы.Цель состоит в том, чтобы записать матрицу \ (A \) с числом \ (1 \) в качестве записи вниз по главной диагонали и иметь все нули сверху и снизу.

\ (A = \ begin {bmatrix} a_ {11} & a_ {12} & a_ {13} \\ a_ {21} & a_ {22} & a_ {23} \\ a_ {31} & a_ {32} & a_ {33} \ end {bmatrix} \ xrightarrow {После \ space Gauss-Jordan \ space elimination} A = \ begin {bmatrix} 1 & 0 & 0 \\ 0 & 1 & 0 \\ 0 & 0 & 1 \ end {bmatrix} \)

Мы можем выполнить операций со строками над матрицей, например сложение, умножение на константу и перестановку строк, чтобы создать сокращенную форму строки-эшелон.Процесс выполнения этих шагов вручную выходит за рамки этого класса. Тем не менее, вы можете найти дополнительную информацию о методе Гаусса-Джордана ЗДЕСЬ.

Решение систем уравнений с исключением Гаусса-Жордана

В рамках этого курса мы продемонстрируем, как найти сокращенную форму строки-эшелон в графическом калькуляторе.

Как: решить систему уравнений с помощью матриц с помощью калькулятора

  1. Сохранить расширенную матрицу как матричную переменную \ ([A], [B], [C] ,… \)
    1. Press 2 nd MATRIX. На экране появится меню «Матрица». Дважды нажмите кнопку со стрелкой вправо, чтобы выбрать меню ПРАВКА. В меню EDIT используйте стрелку вниз для перемещения курсора, чтобы выбрать желаемое имя матрицы из меню, и нажмите ENTER. Появится экран ввода матрицы.

    2. Введите размеры общего размера матрицы в виде строк \ (\ times \) столбцов. Введите количество строк, нажмите ENTER, введите количество столбцов и снова нажмите ENTER.Форма матрицы настраивается на экране, чтобы отобразить требуемое количество строк и столбцов. Убедитесь, что форма соответствует желаемой матрице; в противном случае вернитесь в верхний ряд и отрегулируйте размеры. Если матрица слишком велика для экрана, используйте клавиши со стрелками для прокрутки вправо или вниз, чтобы увидеть оставшиеся строки и столбцы.

    3. Введите элементы матрицы, нажимайте ENTER после каждого. Курсор прокручивает матрицу, перемещаясь по каждой строке слева направо, а затем вниз к следующей строке.Использование клавиш со стрелками для перемещения курсора вместо нажатия ENTER может привести к тому, что значение не будет сохранено в памяти калькулятора.

    4. Нажмите 2 nd QUIT, чтобы завершить процесс сохранения и вернуться на главный экран.

  2. Используйте функцию rref (в калькуляторе, чтобы найти сокращенную форму строки-эшелон матрицы.
    1. На главном экране нажмите 2 nd MATRIX.Используйте стрелку вправо один раз, чтобы перейти в меню МАТЕМАТИКА.

    2. Прокрутите вниз (или вверх) до rref (, стараясь не выбрать ref (, и нажмите ENTER.

    3. Снова нажмите 2 nd MATRIX и используйте стрелку вниз (при необходимости) для выбора имени матрицы и нажмите ENTER.

    4. Нажмите ENTER, чтобы завершить операцию.

  3. Если существует сокращенная форма строки-эшелона матрицы, калькулятор отобразит ее на главном экране. ×

Пример \ (\ PageIndex {4} \): решение систем уравнений с матрицами с помощью калькулятора

Решите систему уравнений.

\ [\ begin {align *} 6x + 4y + 3z & = -6 \\ x + 2y + z & = \ dfrac {1} {3} \\ -12x-10y-7z & = 11 \ end {align *} \ ]

Решение

Напишите расширенную матрицу для системы уравнений.

\ (\ left [\ begin {array} {ccc | c} 6 & 4 & 3 & -6 \\ 1 & 2 & 1 & \ dfrac {1} {3} \\ – 12 & -10 & -7 & 11 \ end {array} \ right] \)

На странице матриц калькулятора введите расширенную матрицу выше как матричную переменную \ ([A] \).

\ ([A] = \ left [\ begin {array} {ccc | c} 6 & 4 & 3 & -6 \\ 1 & 2 & 1 & \ dfrac {1} {3} \\ – 12 & -10 & -7 & 11 \ end {array} \ right] \)

Используйте rref (функция в калькуляторе, вызывая матричную переменную \ ([A] \).

rref ([A])

Используйте опцию MATH -> FRAC в калькуляторе, чтобы выразить матричные элементы в виде дробей.

Оценить

\ [\ begin {array} {cc} {\ left [\ begin {array} {ccc | c} 1 & 0 & 0 & – \ dfrac {2} {3} \\ 0 & 1 & 0 & \ dfrac {5} {2} \\ 0 & 0 & 1 & – 4 \ end {array} \ right] \ rightarrow} & {\ begin {align *} x + 0y + 0z & = – \ dfrac {2} {3} \\ y + 0z & = \ dfrac {5} {2 } \\ z & = -4 \ end {align *}} \ end {array} \]

Таким образом, решение, которое легко найти в правом столбце приведенной строковой формы матрицы, будет \ (\ left (- \ dfrac {2} {3}, \ dfrac {5} {2}, −4 \ справа) \).

Упражнение \ (\ PageIndex {3} \)

Решите систему уравнений.

\ [\ begin {align *} 4x-7y + 2z & = -5 \\ -x + 3y-8z & = -10 \\ -5x-4y + 6z & = 19 \ end {align *} \]

Ответ

Напишите расширенную матрицу для системы уравнений.

\ (\ left [\ begin {array} {ccc | c} 4 & -7 & 2 & -5 \\ -1 & 3 & -8 & -10 \\ -5 & -4 & 6 & 19 \ end {array} \ right] \)

На странице матриц калькулятора введите расширенную матрицу выше как матричную переменную \ ([A] \).

\ ([A] = \ left [\ begin {array} {ccc | c} 4 & -7 & 2 & -5 \\ -1 & 3 & -8 & -10 \\ -5 & -4 & 6 & 19 \ end {array} \ right] \)

Используйте rref (функция в калькуляторе, вызывая матричную переменную \ ([A] \).

rref ([A])

Используйте опцию MATH -> FRAC в калькуляторе, чтобы выразить матричные элементы в виде дробей.

Оценить

\ [\ begin {array} {cc} {\ left [\ begin {array} {ccc | c} 1 & 0 & 0 & -2 \\ 0 & 1 & 0 & 0 \\ 0 & 0 & 1 & \ dfrac {3} {2} \ end {array} \ right] \ rightarrow} & {\ begin {align *} x + 0y + 0z & = -2 \\ y + 0z & = 0 \\ z & = \ dfrac {3} {2} \ end {align *}} \ end {array} \]

Таким образом, решение, которое можно легко прочитать из правого столбца приведенной строковой формы матрицы, будет \ (\ left (-2, 0, \ dfrac {3} {2} \ right) \).

Пример \ (\ PageIndex {5} \): применение матриц \ (2 × 2 \) к финансам

Кэролайн инвестирует в общей сложности \ (12 000 долларов) в две муниципальные облигации, одна из которых выплачивает 10,5% годовых, а другая – 12%. Годовой процент, полученный по двум инвестициям в прошлом году, составил \ (1335 долларов). Сколько было вложено по каждой ставке?

Решение

У нас есть система двух уравнений с двумя переменными. Пусть \ (x = \) сумма, инвестированная под 10,5% годовых, а \ (y = \) сумма, инвестированная под 12%.

\ [\ begin {align *} x + y & = 12,000 \\ 0,105x + 0,12y & = 1,335 \ end {align *} \]

В качестве матрицы имеем

\ (\ left [\ begin {array} {cc | c} 1 & 1 & 12,000 \\ 0.105 & 0.12 & 1335 \ end {array} \ right] \)

Введите эту матрицу как матричную переменную \ ([A] \). Используйте rref (функция , вызывающая матричную переменную \ ([A] \).

rref ([A])

\ (\ left [\ begin {array} {cc | c} 1 & 0 & 7000 \\ 0 & 1 & 5000 \ end {array} \ right] \)

Таким образом, \ ($ 7000 \) было инвестировано по ставке 10.5% годовых и \ (5000 долларов \) под 12% годовых.

Пример \ (\ PageIndex {6} \): применение матриц \ (3 × 3 \) к финансам

Ava инвестирует в общей сложности \ (10 ​​000 долларов США) в три счета, один из которых платит 5% годовых, другой – 8%, а третий – 9%. Годовой процент, полученный по трем инвестициям в прошлом году, составил \ (770 долларов). Сумма, вложенная под 9%, была вдвое больше, чем сумма, вложенная под 5%. Сколько было вложено по каждой ставке?

Решение

У нас есть система трех уравнений с тремя переменными.Пусть \ (x \) будет сумма, инвестированная под 5% годовых, пусть \ (y \) будет суммой, инвестированной под 8%, и пусть \ (z \) будет суммой, инвестированной под 9%. Таким образом,

\ [\ begin {align *} x + y + z & = 10,000 \\ 0,05x + 0,08y + 0,09z & = 770 \\ 2x-z & = 0 \ end {align *} \]

В качестве матрицы имеем

\ (\ left [\ begin {array} {ccc | c} 1 & 1 & 1 & 10,000 \\ 0,05 & 0,08 & 0,09 & 770 \\ 2 & 0 & -1 & 0 \ end {array} \ right] \)

Введите эту матрицу как матричную переменную \ ([A] \).Используйте rref (функция , вызывающая матричную переменную \ ([A] \).

rref ([A])

\ (\ left [\ begin {array} {ccc | c} 1 & 0 & 0 & 0 & 3000 \\ 0 & 1 & 0 & 1000 \\ 0 & 0 & 1 & 6000 \ end {array} \ right] \)

Ответ: \ (3000 долларов \) вложены под 5%, \ (1000 долларов \) вложены под 8%, и \ (6000 долларов \) инвестированы под 9%.

Упражнение \ (\ PageIndex {4} \)

Небольшая обувная компания взяла ссуду в размере \ (1 500 000 долларов США) на расширение своих запасов.Часть денег была взята под 7%, часть – под 8%, часть – под 10%. Сумма займа под 10% в четыре раза превышала сумму займа под 7%, а годовая процентная ставка по всем трем займам составляла \ (130 500 долларов США). Используйте матрицы, чтобы найти сумму, заимствованную по каждой ставке.

Ответ

\ (150 000 долларов \) под 7%, \ (750 000 долларов \) под 8%, \ (600 000 долларов \) под 10%

Медиа

Получите доступ к этим онлайн-ресурсам, чтобы получить дополнительные инструкции и попрактиковаться в решении систем линейных уравнений с использованием метода исключения Гаусса.

Ключевые понятия

  • Расширенная матрица – это матрица, которая содержит коэффициенты и константы системы уравнений. См. Пример \ (\ PageIndex {1} \).
  • Матрица, дополненная постоянным столбцом, может быть представлена ​​как исходная система уравнений. См. Пример \ (\ PageIndex {2} \).
  • Мы можем использовать метод исключения Гаусса-Жордана для решения системы уравнений. См. Пример \ (\ PageIndex {4} \).
  • Многие реальные проблемы можно решить с помощью расширенных матриц.См. Пример \ (\ PageIndex {5} \) и Пример \ (\ PageIndex {6} \).

Авторы и авторство

Исключение по Гауссу – Предварительное вычисление | Сократик

ПРИМЕР:

Используйте метод исключения Гаусса для решения следующей системы уравнений.

# x + 2y + 3z = -7 #
# 2x-3y-5z = 9 #
# -6z-8y + z = -22 #

Решение:

Настроить расширенную матрицу формы.

# ((1,2,3, |, -7), (2,3, -5, |, 9), (- 6, -8,1, |, 22)) #

Цель 1. Получите 1 в верхнем левом углу.

Уже сделано.

Цель 2a: Получите ноль под 1 в первом столбце.

Умножьте строку 1 на # -2 #, чтобы получить

# ((- 2, -4, -6, |, 14)) #

Добавьте результат в строку 2 и поместите результат в строку 2.

Обозначим операции как # -2R_2 + R_1 → R_2 #.

# ((1,2,3, |, -7), (2,3, -5, |, 9), (- 6, -8,1, |, 22)) stackrel (-2R_1 + R_2 → R_2) (→) ((1,2,3, |, -7), (0, -7, -11, |, 23), (- 6, -8,1, |, 22)) #

Цель 2b: Получите еще один ноль в первом столбце.

Для этого нам понадобится операция # 6R_1 + R_3 → R_3 #.

# ((1,2,3, |, -7), (0, -7, -11, |, 23), (- 6, -8,1, |, 22)) stackrel (6R_2 + R_3 → R_3) (→) ((1,2,3, |, -7), (0, -7, -11, |, 23), (0,4,19, |, -64)) #

Цель 2c. Получите оставшийся ноль.

Умножьте строку 2 на # -1 / 7 #.

# ((1,2,3, |, -7), (0, -7, -11, |, 23), (0,4,19, |, -64)) stackrel (- (1/7 ) R_2 → R_2) (→) ((1,2,3, |, -7), (0,1,11 / 7, |, -23 / 7), (0,4,19, |, -64 )) #

Теперь используйте операцию # -4R_2 + R_3 → R_3 #.

# ((1,2,3, |, -7), (0,1,11 / 7, |, -23 / 7), (0,4,19, |, -64)) stackrel (-4R_2 + R_3 → R_3) (→) ((1,2,3, |, -7), (0,1,11 / 7, |, -23 / 7), (0,0,89 / 7, |, -356/7)) #

Умножьте третью строку на # 7/89 #.

# ((1,2,3, |, -7), (0,1,11 / 7, |, -23 / 7), (0,0,89 / 7, |, -356 / 7)) stackrel (7 / 89R_3 → R_3) (→) ((1,2,3, |, -7), (0,1,11 / 7, |, -23 / 7), (0,0,1, | , -4)) #

Цель 3. Используйте обратную замену, чтобы получить значения # x #, # y # и # z #.

Цель 3а. Вычислить # z #.

#z = -4 #

Цель 3b. Вычислить # y #.

# y + 11 / 7z = -23 / 7 #
# y-44/7 = -23 / 7 #
# y = 44 / 7-23 / 7 = 21/7 #

# у = 3 #

Цель 3c. Вычислить x.

# x + 2y + 3z = -7 #
# x + 6-12 = -7 #
# x-6 = -7 #

# х = 1 #

Решение: # x = 1, y = 3, z = -4 #

Я занимаюсь математикой · Программа для одновременного решения линейных уравнений

См. Также: матрица, исключение Гаусса-Жордана, геометрическое линейное преобразование


Калькулятор, представленный ниже, решит одновременные линейные уравнения с двумя, тремя и до 10 переменными, если система уравнений имеет единственное решение.
Для систем уравнений с множеством решений используйте метод исключения Гаусса-Жордана.

Прокрутите вниз, чтобы прочитать о различных методах решения одновременных линейных уравнений.

Программа для решения одновременных линейных уравнений

Выберите размер системы.

загрузка. . .

Расчет. . .


Сообщайте о любых ошибках на [адрес электронной почты]

Методы решения одновременных линейных уравнений

Существует как минимум пять методов решения одновременных линейных уравнений.

Например, давайте попробуем найти решение для следующего набора одновременных линейных уравнений с 3 переменными

{x & plus; y − z = 1 (1) 8⁢x & plus; 3⁢y − 6⁢z = 1 (2) −4⁢x − y & plus; 3⁢z = 1 (3)
Метод исключения

Как следует из названия, этот метод пытается исключить переменные, пока не останется только 1 переменная.

Во-первых, посмотрите на уравнения и попытайтесь найти 2 уравнения с одинаковым коэффициентом (плюс или минус) для одинаковых переменных. Например, см. Уравнения (1) и (3).Коэффициент для y равен 1 и -1 соответственно. Мы можем сложить два уравнения, чтобы исключить y, и мы получим уравнение (4).

x & plus; y − z = 1 (1) −4⁢x − y & plus; 3⁢z = 1 (3) ———————— & plus ; −3⁢x & plus; 0 & plus; 2⁢z = 2 (4)

Обратите внимание, что уравнение (4) состоит из переменных x и z. Теперь нам нужно другое уравнение, которое имеет те же переменные, что и уравнение (4). Чтобы получить это, мы исключим y из уравнений (1) и (2). В уравнениях (1) и (2) коэффициенты при y равны 1 и 3 соответственно.Чтобы исключить y, мы умножаем уравнение (1) на 3, а затем вычитаем уравнение (2) из ​​уравнения (1).

x & plus; y − z = 1 (1) × 3 8⁢x & plus; 3⁢y − 6⁢z = 1 (2)
3⁢x & plus; 3⁢y − 3⁢z = 3 (1) 8⁢x & plus; 3 ⁢Y − 6⁢z = 1 (2) ———————— −−5⁢x & plus; 0⁢y & plus; 3⁢z = 2 (5)

Теперь, используя уравнения (4) и (5), мы можем исключить z.

−3⁢x & plus; 2⁢z = 2 (4) × 3 −5⁢x & plus; 3⁢z = 2 (5) × 2
−9⁢x & plus; 6⁢z = 6 (4) −10⁢x & plus; 6 ⁢Z = 4 (5) ———————— – & plus; 01⁢x & plus; 0⁢z = 2 (6)

Из уравнения ( 6) получаем x = 2.Теперь мы можем подставить это значение x в уравнение (4), чтобы получить значение z.

−3⁢ (2) & plus; 2⁢z = 2 (4) −6 & plus; 2⁢z = 22⁢z = 2 & plus; 62⁢z = 8z = 8 ÷ 2z = 4

Наконец, мы можем подставить значения x и z в уравнение (1), чтобы получить y.

2 & plus; y − 4 = 1 (1) y = 1−2 & plus; 4y = 3

Следовательно, решение системы линейных уравнений имеет вид х = 2, у = 3, г = 4.


Метод замещения

Во-первых, давайте перегруппируем уравнение (1) так, чтобы только 1 переменная находилась в левой части.

x = 1 − y & plus; z (1)

Теперь давайте подставим этот x в уравнение (2).

8⁢ (1 − y & plus; z) & plus; 3⁢y − 6⁢z = 1 (2) 8−8⁢y & plus; 8⁢z & plus; 3⁢y − 6⁢z = 1−5⁢y & plus; 2⁢z = 1−8−5⁢y & plus; 2⁢z = −7 (4)

Аналогичным образом подставим x в уравнение (3).

−4⁢ (1 − y & plus; z) −y & plus; 3⁢z = 1 (3) −4 & plus; 4⁢y − 4⁢z − y & plus; 3⁢z = 13⁢y − z = 1 & plus; 43⁢y− z = 5 (5)

Теперь давайте перегруппируем уравнение (5) так, чтобы только 1 переменная находилась в левой части.

z = 3⁢y − 5 (5)

Затем подставляем это значение z в уравнение (4).

−5⁢y & plus; 2⁢ (3⁢y − 5) = – 7 (4) −5⁢y & plus; 6⁢y − 10 = −7y = −7 & plus; 10y = 3

Теперь, когда мы нашли y, мы можем подставьте это в уравнение (5), чтобы найти z.

z = 3⁢ (3) −5 (5) z = 9−5z = 4

Наконец, мы можем подставить значение y и z в уравнение (1), чтобы получить значение x.

х = 1-3 & плюс; 4 (1) х = 2

Графический метод

Решение системы линейных уравнений с использованием графического метода выполняется путем рисования линий или плоскостей, которые представляют каждое уравнение.Решение – это координаты пересечения линий или плоскостей.

Для простоты рассмотрим систему линейных уравнений с двумя переменными.

{x & plus; y = 32⁢x − y = −3

Постройте линии этих двух уравнений.

Как показано на графике, две прямые пересекаются в точке (0,3). Это решение системы линейных уравнений, т.е. x = 0, у = 3.

Для системы линейных уравнений с тремя переменными решением является точка пересечения трех плоскостей, представляющих каждое уравнение.


Метод обратной матрицы

Система линейных уравнений, определяемая уравнениями (1), (2) и (3), может быть выражена в матричной форме следующим образом.

A⁢B = C12−183−6−4−13⁢xyz = 111

Множеством решений является матрица B. Чтобы выделить только B на одной стороне уравнения, мы умножаем обе части уравнения на матрицу, обратную матрице A

A − 1⁢A⁢B = A − 1⁢CB = A − 1⁢C

Теперь, чтобы найти B, нам нужно найти A − 1. Пожалуйста, проверьте страницу матрицы, чтобы узнать, как найти обратную матрицу.

A − 1 = −323012−435B = −323012−435⁢111B = 234

Следовательно, множество решений х = 2, у = 3, г = 4.


Исключение Гаусса / Исключение Гаусса-Джордана

Система линейных уравнений, определяемая уравнениями (1), (2) и (3), может быть выражена в форме расширенной матрицы следующим образом.

A = 11−1 | 183−6 | 1−4−13 | 1

Выполняя серию операций со строками (исключение Гаусса), мы можем привести указанную выше матрицу к ее эшелонированной форме по строкам.

A = 10,375−0,75 | 0,12501−0,4 | 1.4001 | 4

Затем мы можем выполнить обратную подстановку, чтобы получить значения всех неизвестных / переменных, или мы можем выполнять дальнейшие операции со строками, пока матрица приведена в виде приведенного ряда строк (с использованием метода исключения Гаусса-Жордана).

A = 100 | 2010 | 3001 | 4

Выполняя исключение Гаусса-Жордана, мы получаем решение системы уравнений в последнем столбце: x = 2, у = 3, г = 4.

Чтобы просмотреть пошаговые операции со строками, см. Страницу исключения Гаусса-Джордана.

Запутались, есть вопросы? У нас есть ответы.С Chegg Study вы можете получить пошаговые ответы на свои вопросы от эксперта в этой области.

Джимми Си

См. Также: матрица, исключение Гаусса-Жордана, геометрическое линейное преобразование


Матрицы

и исключение Гаусса

Назад Замена

Напомним, что линейная система уравнений состоит из двух или более линейных уравнений с одинаковыми переменными. Линейная система, состоящая из трех уравнений стандартной формы, расположенных таким образом, что переменная x не появляется ни в одном уравнении после первого, а переменная y не появляется ни в одном уравнении после второго, называется верхнетреугольной формой. линейная система, состоящая из уравнений с тремя переменными в стандартной форме, расположенная так, что переменная x не появляется после первого уравнения, а переменная y не появляется после второго уравнения.. Например,

Обратите внимание, что система образует треугольник, в котором каждое последующее уравнение содержит на одну переменную меньше. В целом

Линейные системы в верхней треугольной форме {a1x + b1y = c1b2y = c2 {a1x + b1y + c1z = d1b2y + c2z = d2c3z = d3

Если линейная система находится в этой форме, мы можем легко найти одну из переменных, а затем произвести обратную замену, чтобы найти оставшиеся переменные.

Пример 1

Решить: {3x − y = 72y = −2.

Решение:

Напомним, что решения линейных систем с двумя переменными, если они существуют, представляют собой упорядоченные пары ( x , y ). Мы можем легко определить значение y , используя второе уравнение.

2у = −2у = −1

Затем используйте первое уравнение 3x − y = 7 и тот факт, что y = −1, чтобы найти x .

3x − y = 73x – (- 1) = 73x + 1 = 73x = 6x = 2

Ответ: (2, −1)

Пример 2

Решите: {x − 6y + 2z = 163y − 9z = 5z = −1.

Решение:

Напомним, что решения линейных систем с тремя переменными, если они существуют, являются упорядоченными тройками ( x , y , z ). Воспользуйтесь вторым уравнением 3y − 9z = 5 и тем фактом, что z = −1, чтобы найти y .

3y − 9z = 53y − 9 (−1) = 53y + 9 = 53y = −4y = −43

Затем подставьте y и z в первое уравнение.

x − 6y + 2z = 16x − 6 (−43) +2 (−1) = 16x + 8−2 = 16x + 6 = 16x = 10

Ответ: (10, −43, −1)

Попробуй! Решить: {4x − y + 3z = 12y − 9z = −23z = 2.

Ответ: (14, 2, 23)

Матрицы и исключение Гаусса

В этом разделе цель – разработать метод, упрощающий процесс решения линейных систем. Мы начинаем с определения матрицы – прямоугольного массива чисел, состоящего из строк и столбцов., Который представляет собой прямоугольный массив чисел, состоящий из строк и столбцов. Для данной линейной системы в стандартной форме мы создаем матрицу коэффициентов Матрицу коэффициентов линейной системы в стандартной форме, записанную так, как они выглядят выстроенной, без переменных или операций.записывая коэффициенты в том виде, в каком они кажутся выстроенными, без переменных или операций, как показано ниже.

Матрица коэффициентов линейной системы {a1x + b1y + c1z = d1a2x + b2y + c2z = d2a3x + b3y + c3z = d3 ⇒ [a1b1c1a2b2c2a3b3c3]

Строки представляют коэффициенты в уравнениях, а столбцы представляют коэффициенты каждой переменной. Кроме того, если мы включим столбец, представляющий константы, мы получим так называемую расширенную матрицу – матрицу коэффициентов с включенным столбцом констант.. Для линейной системы с двумя переменными

Расширенная матрица линейной системы {a1x + b1y = c1a2x + b2y = c2 ⇔ [a1b1 | c1a2b2 | c2]

А для линейной системы с тремя переменными имеем

Расширенная матрица линейной системы {a1x + b1y + c1z = d1a2x + b2y + c2z = d2a3x + b3y + c3z = d3 ⇔ [a1b1c1 | d1a2b2c2 | d2a3b3c3 | d3]

Примечание : Пунктирная вертикальная линия обеспечивает визуальное разделение между матрицей коэффициентов и столбцом констант.В других ресурсах по алгебре, с которыми вы можете столкнуться, это иногда опускается.

Пример 3

Постройте расширенную матрицу, которая соответствует: {9x − 6y = 0 − x + 2y = 1.

Решение:

Эта система состоит из двух линейных уравнений стандартной формы; следовательно, коэффициенты в матрице отображаются так же, как и в системе.

{9x − 6y = 0 − x + 2y = 1 ⇔ [9−6 | 0−12 | 1]

Пример 4

Постройте расширенную матрицу, которая соответствует: {x + 2y − 4z = 52x + y − 6z = 84x − y − 12z = 13.

Решение:

Поскольку уравнения даны в стандартной форме, коэффициенты появляются в матрице так же, как и в системе.

{x + 2y − 4z = 52x + y − 6z = 84x − y − 12z = 13 ⇔ [12−4 | 521−6 | 84−1−12 | 13]

Матрица имеет верхнюю треугольную форму, если все элементы ниже ведущего ненулевого элемента в каждой последующей строке равны нулю. Например,

Обратите внимание, что элементы ниже главной диагонали равны нулю, а коэффициенты выше образуют треугольную форму.В целом

Верхняя треугольная форма [a1b10b2] [a1b1c10b2c200c3]

Это важно, потому что в этом разделе мы очерчиваем процесс, с помощью которого можно выполнить определенные операции для создания эквивалентной линейной системы в верхней треугольной форме, чтобы ее можно было решить с помощью обратной подстановки. Обзор процесса представлен ниже:

Когда система принимает форму верхнего треугольника, мы можем использовать обратную замену, чтобы легко ее решить.Важно отметить, что представленные здесь расширенные матрицы представляют собой линейные системы уравнений в стандартной форме.

Следующие элементарные операции со строками Операции, которые могут быть выполнены для получения эквивалентных линейных систем. приводят к расширенным матрицам, которые представляют эквивалентные линейные системы:

  1. Любые две строки можно поменять местами.
  2. Каждый элемент в строке можно умножить на ненулевую константу.
  3. Любая строка может быть заменена суммой этой строки и кратной другой.

Примечание: Эти операции согласуются со свойствами, используемыми в методе исключения.

Чтобы эффективно решить систему линейных уравнений, сначала постройте расширенную матрицу. Затем примените соответствующие элементарные операции со строками, чтобы получить расширенную матрицу в форме верхнего треугольника. В этой форме эквивалентная линейная система может быть легко решена с помощью обратной подстановки. Этот процесс называется гауссовским устранением. Шаги, используемые для получения эквивалентной линейной системы в верхней треугольной форме, чтобы ее можно было решить с помощью обратной подстановки., названный в честь Карла Фридриха Гаусса (1777–1855).

Рисунок 3.1

Карл Фридрих Гаусс (Википедия)

Шаги решения линейного уравнения с двумя переменными с использованием исключения Гаусса перечислены в следующем примере.

Пример 5

Решить, используя матрицы и метод исключения Гаусса: {9x − 6y = 0 − x + 2y = 1.

Решение:

Перед началом этого процесса убедитесь, что уравнения в системе имеют стандартную форму.

Шаг 1 : Постройте соответствующую расширенную матрицу.

{9x − 6y = 0 − x + 2y = 1 ⇔ [9−6 | 0−12 | 1]

Шаг 2 : Примените операции элементарной строки для получения верхней треугольной формы. В этом случае нам нужно только удалить первый элемент второй строки, −1. Для этого умножьте вторую строку на 9 и прибавьте ее к первой строке.

Теперь используйте это, чтобы заменить вторую строку.

[9−6 | 0012 | 9]

В результате получается расширенная матрица в форме верхнего треугольника.

Шаг 3 : Преобразуйте обратно к линейной системе и решите, используя обратную подстановку. В этом примере у нас

[9−6 | 0012 | 9] ⇒ {9x − 6y = 012y = 9

Решите второе уравнение относительно y ,

12y = 9y = 912y = 34

Подставьте это значение вместо y в первое уравнение, чтобы найти x ,

9x − 6y = 09x − 6 (34) = 09x − 92 = 09x = 92x = 12

Ответ: (12, 34)

Шаги по использованию исключения Гаусса для решения линейного уравнения с тремя переменными перечислены в следующем примере.

Пример 6

Решить, используя матрицы и метод исключения Гаусса: {x + 2y − 4z = 52x + y − 6z = 84x − y − 12z = 13.

Решение:

Перед началом этого процесса убедитесь, что уравнения в системе имеют стандартную форму.

Шаг 1 : Постройте соответствующую расширенную матрицу.

{x + 2y − 4z = 52x + y − 6z = 84x − y − 12z = 13 ⇒ [12−4 | 521−6 | 84−1−12 | 13]

Шаг 2 : Примените операции элементарной строки для получения верхней треугольной формы.Начнем с исключения первого элемента второй строки, в данном случае 2. Для этого умножьте первую строку на −2, а затем добавьте ее во вторую строку.

[12−4 | 521−6 | 84−1−12 | 13] ⇒ × (−2) −2−48−10 + 21−680−32−2

Используйте это, чтобы заменить вторую строку.

[12−4 | 50−32 | −24−1−12 | 13]

Затем удалите первый элемент третьей строки, в данном случае 4, умножив первую строку на −4 и прибавив ее к третьей строке.

[12−4 | 50−32 | −24−1−12 | 13] ⇒ × (−4) −4−816−20 + 4−1−12130−94−7

Используйте это для замены третьей строки.

[12−4 | 50−32 | −20−94 | −7]

В результате получается расширенная матрица, в которой элементы под первым элементом первой строки равны нулю. Затем удалите второй элемент в третьей строке, в данном случае −9. Умножьте вторую строку на −3 и прибавьте ее к третьей строке.

Используйте это, чтобы заменить третью строку, и мы видим, что мы получили матрицу в форме верхнего треугольника.

[12−4 | 50−32 | −200−2 | −1]

Шаг 3 : Преобразуйте обратно к линейной системе и решите, используя обратную подстановку. В этом примере у нас

[12−4 | 50−32 | −200−2 | −1] ⇒ {x + 2y − 4z = 5−3y + 2z = −2−2z = −1

Ответ: Читателю остается убедиться, что решение (5,1,12).

Примечание: Обычно работа по замене строки путем умножения и сложения выполняется на стороне с использованием бумаги для заметок.

Пример 7

Решить, используя матрицы и метод исключения Гаусса: {2x − 9y + 3z = −18x − 2y − 3z = −8−4x + 23y + 12z = 47.

Решение:

Начнем с преобразования системы в расширенную матрицу коэффициентов.

{2x − 9y + 3z = −18x − 2y − 3z = −8−4x + 23y + 12z = 47 ⇒ [2−93 | −181−2−3 | −8−42312 | 47]

Операции с элементарными строками упрощаются, если ведущий ненулевой элемент в строке равен 1.По этой причине начните с того, что поменяйте местами первый и второй ряды.

Заменить вторую строку суммой −2, умноженной на первую и вторую строку.

Заменить третью строку суммой четырех строк первой и третьей.

Далее разделите 3-ю строку на 15.

Поменяйте местами третий ряд со вторым.

Затем замените строку 3 суммой, умноженной на 5 строк второй и третьей.

В результате получается матрица в форме верхнего треугольника. Матрица находится в виде эшелона строк Матрица в треугольной форме, где ведущий ненулевой элемент каждой строки равен 1., если она находится в верхней треугольной форме, где ведущий ненулевой элемент каждой строки равен 1. Мы можем получить эту форму, заменив третью строку на результат деления на 9.

Преобразуйте в систему линейных уравнений и решите обратной подстановкой.

[1−2−3 | −8010 | 1001 | 13] ⇒ {x − 2y − 3z = −8y = 1z = 13

Здесь y = 1 и z = 13. Подставляем в первое уравнение, чтобы найти x .

x − 2y − 3y = −8x − 2 (1) −3 (13) = – 8x − 2−1 = −8x − 3 = −8x = −5

Ответ: Следовательно, решение – (−5, 1, 13).

Технологическое примечание : Многие современные калькуляторы и системы компьютерной алгебры могут выполнять метод исключения Гаусса. Сначала вам нужно узнать, как войти в матрицу.Затем используйте функции калькулятора, чтобы найти форму эшелона строки. Предлагаем вам провести исследование по этой теме для вашей конкретной модели калькулятора.

Попробуй! Решить, используя исключение Гаусса: {x − 3y + 2z = 164x − 11y − z = 692x − 5y − 4z = 36.

Ответ: (6, −4, −1)

Напомним, что некоторые непротиворечивые линейные системы зависимы, то есть у них бесконечно много решений.А некоторые линейные системы не имеют одновременного решения; это несовместимые системы.

Пример 8

Решить, используя матрицы и метод исключения Гаусса: {x − 2y + z = 42x − 3y + 4z = 74x − 7y + 6z = 15.

Решение:

Начнем с преобразования системы в расширенную матрицу коэффициентов.

{x − 2y + z = 42x − 3y + 4z = 74x − 7y + 6z = 15 ⇒ [1−21 | 42−34 | 74−76 | 15]

Заменить строку два на −2 (строка 1) + (строка 2) и заменить строку три на −4 (строка 1) + (строка 3).

[1−21 | 4012 | −1012 | −1]

Заменить третью строку на −1 (строка 2) + (строка 3).

[1-21 | 4012 | -1000 | 0]

Последняя строка указывает, что это зависимая система, потому что преобразование расширенной матрицы обратно в уравнения, которые у нас есть,

{x − 2y + z = 4y + 2z = −10x + 0y + 0z = 0

Обратите внимание, что строка нулей соответствует следующему тождеству,

0x + 0y + 0z = 00 = 0 ✓

В этом случае мы можем выразить бесконечное множество решений через z .Из второго ряда имеем:

y + 2z = −1y = −2z − 1

И из первого уравнения,

x − 2y + z = 4x − 2 (−2z − 1) + z = 4x + 5z + 2 = 4x = −5z + 2

Решения имеют вид (x, y, z) = (- 5z + 2, −2z − 1, z), где z – любое действительное число.

Ответ: (−5z + 2, −2z − 1, z)

Зависимые и несовместимые системы могут быть идентифицированы в расширенной матрице коэффициентов, когда все коэффициенты в одной строке равны нулю.

Если строка нулей имеет соответствующую константу, равную нулю, тогда матрица представляет зависимую систему. Если константа отлична от нуля, матрица представляет собой несовместимую систему.

Попробуй! Решить, используя матрицы и метод исключения Гаусса: {5x − 2y + z = −310x − y + 3z = 0−15x + 9y − 2z = 17.

Ответ: Ø

Ключевые выводы

  • Линейная система в верхней треугольной форме может быть легко решена с помощью обратной подстановки.
  • Расширенная матрица коэффициентов и метод исключения Гаусса могут использоваться для упрощения процесса решения линейных систем.
  • Чтобы решить систему с использованием матриц и исключения Гаусса, сначала используйте коэффициенты для создания расширенной матрицы. Примените операции с элементарными строками как средство для получения матрицы в форме верхнего треугольника. Преобразуйте матрицу обратно в эквивалентную линейную систему и решите ее, используя обратную подстановку.

Тематические упражнения

    Часть A: Назад Замена

      Решите, используя обратную замену.

    1. {5x − 3y = 2y = −1

    2. {3x + 2y = 1y = 3

    3. {x − 4y = 12y = −3

    4. {x − 5y = 310y = −6

    5. {4x − 3y = −167y = 0

    6. {3x − 5y = −104y = 8

    7. {2x + 3y = −13y = 2

    8. {6x − y = −34y = 3

    9. {х-у = 02у = 0

    10. {2x + y = 23y = 0

    11. {x + 3y − 4z = 1y − 3z = −2z = 3

    12. {x − 5y + 4z = −1y − 7z = 10z = −2

    13. {x − 6y + 8z = 23y − 4z = −42z = −1

    14. {2x − y + 3z = −92y + 6z = −23z = 2

    15. {10x − 3y + z = 1311y − 3z = 92z = −6

    16. {3x − 2y + 5z = −244y + 5z = 34z = −12

    17. {x − y + 2z = 12y + z = 13z = −1

    18. {x + 2y − z = 2y − 3z = 16z = 1

    19. {x − 9y + 5z = −32y = 103z = 27

    20. {4x – z = 33y − 2z = −12z = −8

    Часть B: Матрицы и исключение Гаусса

      Построить соответствующую расширенную матрицу (не решать).

    1. {х + 2у = 34х + 5у = ​​6

    2. {6x + 5y = 43x + 2y = 1

    3. {x − 2y = 12x − y = 1

    4. {х-у = 2-х + у = -1

    5. {−x + 8y = 32y = 2

    6. {3x − 2y = 4 − y = 5

    7. {3x − 2y + 7z = 84x − 5y − 10z = 6 − x − 3y + 2z = −1

    8. {x − y − z = 02x − y + 3z = −1 − x + 4y − 3z = −2

    9. {x − 9y + 5z = −32y = 103z = 27

    10. {4x − z = 33y − 2z = −12z = −8

    11. {8x + 2y = −13−2y + z = 112x − 5z = −18

    12. {x − 3z = 2y + 6z = 42x + 3y = 12

      Решите, используя матрицы и метод исключения Гаусса.

    1. {x − 5y = 22x − y = 1

    2. {x − 2y = −1x + y = 1

    3. {10x − 7y = 15−2x + 3y = −3

    4. {9x − 10y = 23x + 5y = −1

    5. {3x + 5y = 82x − 3y = 18

    6. {5x − 3y = −147x + 2y = −1

    7. {9x + 15y = 53x + 5y = 7

    8. {6x − 8y = 1−3x + 4y = −1

    9. {х + у = 0х-у = 0

    10. {7x − 3y = 03x − 7y = 0

    11. {2x − 3y = 4−10x + 15y = −20

    12. {6x − 10y = 20−3x + 5y = −10

    13. {x + y − 2z = −1 − x + 2y − z = 1x − y + z = 2

    14. {x − y + z = −2x + 2y − z = 6 − x + y − 2z = 3

    15. {2x − y + z = 2x − y + z = 2−2x + 2y − z = −1

    16. {3x − y + 2z = 7 − x + 2y + z = 6x + 3y − 2z = 1

    17. {x − 3y + z = 6 − x − y + 2z = 42x + y + z = 3

    18. {4x − y + 2z = 12x − 3y + 2z = 7−2x + 3y + 4z = −16

    19. {2x − 4y + 6z = −43x − 2y + 5z = −25x − y + 2z = 1

    20. {3x + 6y + 9z = 62x − 2y + 3z = 0−3x + 18y − 12z = 5

    21. {−x + y − z = −23x − 2y + 5z = 13x − 5y − z = 3

    22. {x + 2y + 3z = 43x + 8y + 13z = 212x + 5y + 8z = 16

    23. {2x − 4y − 5z = 3 − x + y + z = 13x − 4y − 5z = −4

    24. {5x − 3y − 2z = 43x − 6y + 4z = −6 − x + 2y − z = 2

    25. {−2x − 3y + 12z = 44x − 5y − 10z = −1 − x − 3y + 2z = 0

    26. {3x − 2y + 5z = 104x + 3y − 3z = −6x + y + z = 2

    27. {x + 2y + z = −3x + 6y + 3z = 7x + 4y + 2z = 2

    28. {2x − y + z = 14x − y + 3z = 52x + y + 3z = 7

    29. {2x + 3y − 4z = 03x − 5y + 3z = −105x − 2y + 5z = −4

    30. {3x − 2y + 9z = 2−2x − 5y − 4z = 35x − 3y + 3z = 15

    31. {8x + 2y = −13−2y + z = 112x − 5z = −18

    32. {x − 3z = 2y + 6z = 42x + 3y = 12

    33. {9x + 3y − 11z = 62x + y − 3z = 17x + 2y − 8z = 3

    34. {3x − y − z = 4−5x + y + 2z = −36x − 2y − 2z = 8

    35. {2x − 4y + 3z = 153x − 5y + 2z = 185x + 2y − 6z = 0

    36. {3x − 4y − 3z = −144x + 2y + 5z = 12−5x + 8y − 4z = −3

    Часть C: Обсуждение

    1. Изучите и обсудите историю метода исключения Гаусса.Кто первым разработал этот процесс? Опубликуйте что-нибудь, что вам показалось интересным в связи с этой историей.

    2. Изучите и обсудите историю современной матричной записи. Кому засчитывается разработка? В каких сферах они используются сегодня? Разместите свои выводы на доске обсуждений.

ответов

  1. (-15, -1)

  2. (-5, -32)

  3. (-32,23)

  4. (−6, −2, −12)

  5. (85,0, −3)

  6. (73,23, −13)

  1. [12 | 345 | 6]

  2. [1-2 | 12-1 | 1]

  3. [−18 | 302 | 2]

  4. [3−27 | 84−5−10 | 6−1−32 | −1]

  5. [1−95 | −3020 | 10003 | 27]

  6. [820 | −130−21 | 1120−5 | −18]

  7. (13, −13)

  8. (32,0)

  9. (х, 23x − 43)

  10. (12,12, −12)

  11. (1,0,12)

  12. (−8, −12z + 52, z)

  13. (-32, -12, 0)

систем линейных уравнений: исключение Гаусса

систем линейных уравнений: исключение Гаусса

Решать нелинейные системы уравнений довольно сложно, а линейные системы довольно легко изучать.Существуют численные методы, которые помогают аппроксимировать нелинейные системы линейными в надежде, что решения линейных систем достаточно близки к решениям нелинейных систем. Мы не будем здесь обсуждать это. Вместо этого мы сосредоточим наше внимание на линейных системах.

Для простоты мы ограничимся тремя, максимум четырьмя неизвестными. Читатель, интересующийся случаем большего количества неизвестных, может легко развить следующие идеи.

Определение. Уравнение

a x + b y + c z + d w = h


где a , b , c , d и h – известные числа, а x , y , z и w – неизвестные числа. называется линейным уравнением . Если h = 0, линейное уравнение называется однородным . Линейная система , – это набор линейных уравнений, а однородная линейная система , – это набор однородных линейных уравнений.

Например,


а также

линейные системы, а

является нелинейной системой (из-за y 2 ). Система

является однородной линейной системой.

Матричное представление линейной системы

Матрицы помогают переписать линейную систему в очень простой форме. Затем для решения систем можно использовать алгебраические свойства матриц. Сначала рассмотрим линейную систему


Установите матрицы

Используя матричное умножение, мы можем переписать линейную систему выше как матричное уравнение

Как видите, это намного лучше, чем уравнения.Но иногда стоит решить систему напрямую, минуя матричную форму. Матрица A называется матричным коэффициентом линейной системы. Матрица C называется неоднородным членом . Когда , линейная система однородна. Матрица X – это неизвестная матрица. Его записи являются неизвестными линейной системы. Расширенная матрица , связанная с системой, является матрицей [ A | С ], где

В общем, если линейная система имеет n уравнений с m неизвестными, то матричный коэффициент будет матрицей nxm, а расширенная матрица – матрицей nx (m + 1).Теперь обратим внимание на решения системы.

Определение. Две линейные системы с n неизвестными считаются эквивалентами тогда и только тогда, когда они имеют одинаковый набор решений.

Это определение важно, поскольку идея решения системы состоит в том, чтобы найти эквивалентную систему, которую легко решить. Вы можете спросить, как мы сможем создать такую ​​систему? Легко, мы делаем это с помощью элементарных операций . Действительно, ясно, что если мы поменяем местами два уравнения, новая система все равно будет эквивалентна старой.Если мы умножим уравнение на ненулевое число, мы получим новую систему, по-прежнему эквивалентную старой. И, наконец, заменив одно уравнение суммой двух уравнений, мы снова получим эквивалентную систему. Эти операции называются элементарными операциями в системе. Посмотрим, как это работает в конкретном случае.

Пример. Рассмотрим линейную систему

Идея состоит в том, чтобы сохранить первое уравнение и поработать над двумя последними. При этом мы попытаемся убить одного из неизвестных и решить два других.Например, если мы сохраним первое и второе уравнение и вычтем первое из последнего, мы получим эквивалентную систему


Затем мы сохраняем первое и последнее уравнение и вычитаем первое из второго. Получаем эквивалентную систему

Теперь мы сосредоточимся на втором и третьем уравнениях. Повторяем ту же процедуру. Попробуйте убить одного из двух неизвестных ( y или z ). Действительно, мы сохраняем первое и второе уравнение и добавляем второе к третьему, умножив его на 3.Мы получили

Это, очевидно, означает z = -2. Из второго уравнения мы получаем y = -2, и, наконец, из первого уравнения получаем x = 4. Следовательно, линейная система имеет одно решение.

Переход от последнего уравнения к первому при решении неизвестных называется обратным решением .

Имейте в виду, что линейные системы, для которых матричный коэффициент является верхнетреугольным, легко решить. Это особенно верно, если матрица имеет эшелонированную форму.Таким образом, фокус состоит в том, чтобы выполнить элементарные операции по преобразованию исходной линейной системы в другую, для которой матрица коэффициентов имеет эшелонированную форму.
Используя наши знания о матрицах, можем ли мы в любом случае переписать то, что мы сделали выше, в матричной форме, которая упростит нашу нотацию (или представление)? Действительно, рассмотрим расширенную матрицу


Выполним над этой матрицей несколько элементарных операций со строками. Действительно, если мы сохраним первую и вторую строки и вычтем первую из последней, мы получим

Затем мы сохраняем первую и последнюю строки и вычитаем первую из второй.Мы получили

Затем мы сохраняем первую и вторую строки и добавляем вторую к третьей, умножив ее на 3, чтобы получить

Это треугольная матрица, не имеющая эшелонированной формы. Линейная система, для которой эта матрица является расширенной, есть

Как видите, мы получили ту же систему, что и раньше. Фактически мы следовали тем же элементарным операциям, что и выше. На каждом этапе новая матрица была в точности расширенной матрицей, связанной с новой системой.Это показывает, что вместо того, чтобы писать системы снова и снова, легко поиграться с элементарными операциями со строками, и как только мы получим треугольную матрицу, напишем связанную линейную систему, а затем решим ее. Это известно как Гауссовское исключение . Подведем итоги процедуры:

Исключение по Гауссу. Рассмотрим линейную систему.

1.
Построить расширенную матрицу для системы;
2.
Используйте элементарные операции со строками, чтобы преобразовать расширенную матрицу в треугольную;
3.
Запишите новую линейную систему, для которой треугольная матрица является связанной с ней расширенной матрицей;
4.
Решите новую систему. Вам может потребоваться присвоить некоторые параметрические значения некоторым неизвестным, а затем применить метод обратной подстановки для решения новой системы.

Пример. Решите следующую систему методом исключения Гаусса.


Расширенная матрица

Мы используем элементарные операции со строками, чтобы преобразовать эту матрицу в треугольную.Мы сохраняем первую строку и используем ее для получения всех нулей в любом месте первого столбца. У нас есть

Далее мы сохраняем первую и вторую строки и стараемся, чтобы во втором столбце были нули. Мы получили

Далее сохраняем первые три ряда. Добавляем последний к третьему, чтобы получить

Это треугольная матрица. Связанная с ним система

Очевидно, что v = 1. Положим z = s и w = t , тогда получим

Из первого уравнения следует Используя алгебраические манипуляции, получаем
x = – – с т .
Собрав все вместе, у нас есть

Пример. Используйте метод исключения Гаусса для решения линейной системы


Соответствующая расширенная матрица

Сохраняем первую строку и вычитаем первую строку, умноженную на 2, из второй строки. Мы получили

Это треугольная матрица. Связанная система

Ясно, что второе уравнение означает, что эта система не имеет решения. Следовательно, эта линейная система не имеет решения.

Определение. Линейная система называется несовместимой или переопределенной , если у нее нет решения. Другими словами, набор решений пуст. В противном случае линейная система называется согласованная .

Следуя приведенному выше примеру, мы видим, что если мы выполним элементарные операции со строками над расширенной матрицей системы и получим матрицу с одной из строк, равной , где , тогда система несовместима.

[Назад] [Следующий] [Геометрия] [Алгебра] [Тригонометрия] [Исчисление] [Дифференциальные уравнения] [Матричная алгебра]

С.O.S MATH: Домашняя страница

Вам нужна дополнительная помощь? Пожалуйста, разместите свой вопрос на нашем S.O.S. Математика CyberBoard.

Автор : М.А.Хамси

Авторские права 1999-2021 MathMedics, LLC.

Добавить комментарий

Ваш адрес email не будет опубликован. Обязательные поля помечены *